SlideShare a Scribd company logo
1 of 51
Download to read offline
FE Exam Review for
Structural Analysis
Prof. V. Saouma
Oct. 2013
 Structural Analysis is part of the afternoon exam.
 In the afternoon, you are to answer 60 questions, and Structural Analysis is about 10%
of the test content (or about 6 questions).
 Each question is worth 2 points.
 You are expected to know:
1. Structural analysis of statically determinate beams, trusses and frames.
2. Deflection analysis of statically determinate beams, trusses and frames.
3. Stability analysis of beams, trusses and frames.
4. Column analysis (e.g. buckling, boundary conditions).
5. Loads and load paths (e.g. dead, live, moving).
6. Elementary statically indeterminate structures.
 The only page in the “Supplied-Reference Handbook” related to Structural Analysis
(shown in the next page).
 Make sure that you know how to make best use of it, as it contains:
1. Reminder of what do we mean by “Moving Loads”.
2. Beam-Stiffness and moment carryover: to use for the analysis of statically
indeterminate beams (unlikely that you get a SI frame).
3. Equations for the calculations of the deflections of trusses and beams using the
virtual work method. Careful it is the virtual force/moment time the actual
displacement (FL/AE for trusses, and M/EI for beams).
4. Member fixed end actions for uniform and concentrated load.
 I strongly recommend that you also memorize: for the maximum deflection of
a uniformly loaded, simply supported beam.
 Careful with the SI units, GPa is 109
Pa or 109
N/m2
Many problems use the SI system.
 In most cases, you will be dealing with round numbers, which greatly simplify your
calculations.
 Do not be tricked in believing that all triangles are 3-4-5.
26 (ivilDisdpline回 SpecificReviewfortheFE/EITEx目m
FromthePVC,thelowpointislocatedat
mA
吐
A
せ十
m
一
m
的
町
一
つG
A
斗
A
F
L一
A
吐
0
0一
A
せ
一一一一
mA
せ
4
パヨ+
L
一2
Determinethetangentoffsetぅ仏 atthelowpoint.
y=iG2- G1)X
2
2L
- (0.016一 (-0.02))(444m)2
(2)(800m)
ニ 4.44m
eleVlowpoint=742.12m十 4.44m
=746.56m (747m)
TheanswerisB.
STRUCTURALANA町SIS
Problem38
They-coordinate(mea師su町1
tむroぱidfortheT-shapedbeamis
「一「ーr---I
J08m比
X
y
0.23m
(A) 0.08m
(B) 0.16m
(C)0却 m
(D) 0.38m
PPI・ 附W押i2pllss.com
Solution
Divide七hebeamin七otwoshapesasshown
Al=(0.08m)(0.15m)
=0.012m2
A2=(0.05m)(0.15m)
ェ 0.0075m2
Thedis七ancefrom七hetopofsection2tothecentroidis
Uc=Z生Yc,i
2乞:Aん4
(( 川
十((0.0075m
2
)(平十0叫))
ニ 0.08m
TheanswerisA.
Problem39
TheforceinmemberHEfromthe七russshownismos七
nearly
G
~1
12.0m
(A) 1110Ncompression
(B) 1110Ntension
(C) 2490Ncompression
(D) 2490N七ension
Solved Problems
Solution
8ummomentsaboutAtofindtheverticalreactionatE.
:Z=MA= 0
RE,匂
dEA
RH:"=i2224N)(伊6m)
U円'げν 1ロ2m
=1112N
Thefree-bodydiagramabou七pointEis
、....._.EH
DE !RE,ν
8ummingforcesinthehorizontaldirectiongivesthe
horizontalreactionatEas0N.
Thesumofforcesin位1everticaldirectionis
FEH,υ-RE,り=0lbf
=1112Ncompression
ThehorizontalcomponentoftheforceinmemberEHis
FEH,h=(2)(1112N)
=2224N
Theresultan七forceinmemberEHis
FEH=V(Fi凹,出(Fi叫2
=ゾ(1112N)2十 ( 山 N)2
ニ 2487Ncompre回 on (2490N∞mpr出品on)
TheanswerisC.
Problems40-42arebasedonthefollowinginformation
andillustra七ion.
Aplanetrussisloadedasshown.
Pr目di(eProblems 27
Problem40
Themagnitudeoftheverticalreactionforceatsup-
portAismostnearly
(A) 3.3kN
(B) 6.7kN
(0) 10kN
(D)16kN
Solution
8incesupportDisarollersupportう thehorizontalre-
ac七ionforceう RAx,is0kN.Tofindtheverticalreaction
atsupportA,RAy,afree-bodydiagramisdrawnofthe
entire七russandmomentsaresummedaboutsuppor七D.
RAy(15m)-(20kN)(5m)=0kN
RA"=i20kN)(5m)
吋 15m
=6.67kN (6.7kN)
TheanswerisB.
Problem41
ThemagnitudeofthecompressiveforceinmemberAB
ismostnearly
(A)4.2kN
(B) 6.7凶
(0) 8.5kN
(D) 11kN
Solution
From801.40,thereactionatsupportA is6.67kN.
Nextぅafree同 bodydiagr乱misdrawnforsupportAwith
memberforc田 and七heirforcecomponents.Using七he
Pythagoreantheorem,therelativemagnitudesofeach
forceandeachforceうshorizontalandverticalcompo-
m臨 canbefound. (Inthiscase,6.403,5,and4are
therelativemagni七udesofthememberABforceand
theforcesうshorizontalandverむicalcomponen七s,respec-
tively.)
FAEx
PPI.附 w.ppi2p日sS.com
28 CivilDiscipline-Specifi(ReviewforfheFE/EITEx目m
Forequilibrium,allforcesonafreebodymus七sumto
okN.Summationofverticalforcesgiv<回
FABy+RAy=0kN
Thiscanberearrangedtogive
FABy= -RAy=-6.67kN
Recallthatjointsin七russesarefrictionless,sonobend-
ingmomentsexist.
Theforceanditscomponentsareproportionaltothe
geometriclengもhsofthetrianglesides.
一 (6.403m¥一
.l'AB= I一一了一一一一 I.l'AB'U
¥ 4m I ν
(6.403m¥
=(τ'~--- )(-6.67kN)
= -10.68kN (-11kN)
Theanswerisnegative.Thismeansthatthecalculated
forceisintheopposi七edirec七ionto七heassumedforcedi-
rectiononthefree-bodydiagram.8incethememberAB
forcew回 assumedtoapplytensiononthefree伊 bodydi-
agram,thenegativeanswermeansthatmemberABis
mcompreSSlOn.
TheanswerisD.
Problem42
ThemagnitudeoftheforceinmemberEFismos七ne乱rly
(A)4.2kN
(B) 5.3kN
(C) 6.7kN
(D)8.3悶
Solution
From801.41,FAByis-6.67 日~. Thehorizontalcom-
ponentofthememberABforceis
M
H
L
且
可
tn
h
v
U
F
O
E
U
ρ
h
u
凡
ト
N
¥
1
1
1
1
/
¥、
1
1
1
I
/
L
ι
m
一
m
m
一
m
お
v
h
u
一A
住
民
U
一an笹
口
δ
/
e
l
l
-
¥
/
1
1
1
1
¥
一
一
一
一
一
一
一
ZBAF
ForequilibriumatsupportA,thesumofthehorizontal
forcesmustbeequalto0kN.TheforceinmemberAEis
FAE=FAEx
= -FABx
=-(-8.33kN)
=8.33kN
PPI.附 W押i2p日ss.com
Theposi七ivesigninthecalculatedmemberAEforce
meansthattheassumeddirectionoftheforceon七he
free-bodydiagramう whichindicatestension,isinthe
samedirectionasthecalculatedforce.
Afree-bodydiagramofjointEwillshowthatthever-
ticalmemberBEisunabletosustainanyhorizontal
force. Therefore,theforceinmember~F isthesame
astheforceinmemberAE.
FEF=FAE
=8.33kN (8.3kN)
TheanswerisD.
Problem43
Ifthetrussmembersaremadeofsteelandthecross-
sectionalareaofeachmemberis1000mm2
,themagni-
tudeoftheverticaldefl.ection抗 jointEismostnearly
(A) 0.70mm
(B) 1.1mm
(C) 1.6m m
(D)2.8mm
Solution
Usetheprincipleofvi巾 alwork(unitloadmethod).
Theactualforcesineachmembercanbedetermined
byapplyingtheequationsofequilibriumtoeachtruss
jointandareshown回 follows. (80meround-offerror
existsinthesecalculatednumbers.)
τ'russmemberlengthsinmetersareasfollows.
B 5 C
AT< 41 々ず'" 1
4 仏
/ I
、 士 一 '::"'D
5 E 5 F
Theactualforcesinkilonewtonsareshown.
20
D
6.67 13.33
PradiceProblems 29
Problems44and45arebasedonthefollowingillustra司
tion.
Applicationofaverticalunitloadatjoi凶 E results
inthevirtualmemberforcesinkilonewtonsasfollows.
(Someroundoffeロorexis匂 inthesecalculatednum-
bers.)
町5m ↑ 5m 手5m JD
Problem44
Themagnitudeofthever七icalreac七ionforceatsup-
port.Aismostnearly
0.33
Itisrecommendedthatatablebeusedtokeepallvari-
ablesorganized.Usenegativevaluestorepresentcom-
presslOn.
0.67
1.3kN
5.0kN
13kN
20kN
(A)
(B)
(C)
(D)
FQFpL
(kN2
岨)
L
(m)
FQ
(kN)
Fp
(kN)
Solution
Sincesuppor七Cisarollersuppor七ぅ thereisnohorizon-
talreactionforceatthatpoint. Theverticalreaction
forceatsupportAう RA約 canbefoundbyconverting
theuniformlydistributedload,ω,intoaresultantpoint
load.
mv
h
u
¥、、
E
B﹄
t
/ノ
凶一
m
v
h
u
/
I
I
I
-、¥
二
到
L
i
ω
引制
一
一
一
一
W
member
73.1
35.0
72.4
34.6
34.6
35.0
0.0
8.8
-36.2
I:=239.7
6.4
5.0
6.4
5.0
5.0
5.0
4.0
4.0
6.4
-1.07
-0.42
0.53
+0.83
+0.83
十0.42
+1.0。+0.33
-0.53
10.67
-16.67
-21.34
十8.33
十8.33
+16.67
十0.00
-6.67
+10.67
B
C
D
E
F
D
E
F
F
A
B
C
A
E
F
B
C
B
Thisresul七antloadislocateda七thecentroidoftheuni-
formlydistributedload.
ThemodulusofelasticityofsteelisE=2.1X 1011
Pa.
Sincetheareaandmodulusofelasticityarethesame
foralltrussmembers,七heirproduct,AEう iscommonto
allmembersandcanbetakenoutsideofthesummation
forsimpli五cation. Thereforβ,theverticaldeflectionat
pointEcanbefoundby
r山
一
E
H
一A
?
Q
L
E
A
い
P
ε
v
b
z
M
¥
l
/
R
l
一E
Z
仁
い
EA
5m
ThesupportAverticalreactioncanthenbefoundby
summingmomentsaboutsupportC.Thisresultsina
verticalreactionatsupportAof
5m(1000mm
2
)(吋:m)
× 川1Pa)(品)
RAy(10m)-(15kN)(5m)
+(25kN)(2.5m)=0kN-m
x(1000誓)
(1.1mr吋
ISincetheunitloadinthevirtualforcesystemwasdown-
.wardandtheanswerispositiveinsign,theactualde-
丑ectionisalsodownward. RA"ニ (15kN)(5m)-(25kN)(2.5m)
吋 10m
(1.3kN)
TheanswerisB.
PPI.附 W押i2p日sS.com
= 1.25kN
ェ1.14rnrn
30 CivilDiscipline-SpecificReviewfortheFE/EITExam
Sincetheanswerispositiveinsign,thedirec七ionofthe
calculatedreactionisthesameasthatoftheassumed
reactionithatis,thedirectionofthereactionisupw乱rd.
TheanswerisA.
Problem45
Themagnitudeofthemaximumverticalshearinthe
beamismostnearly
(A) 1.3kN
(B) 14kN
(C) 25kN
(D) 39kN
Solution
Oneway七odeterminetheanswertothisproblemisto
constructasheardiagram.Thechangeinshearisthe
areaundertheappliedloading. Althoughamomen土
diagramisnotrequiredforthisproblemう itfollowsthat
thechangeinmomen七istheareaunderthesheardia-
gram,soamomentdiagramisusuallyincluded.
/ llF15kN LkN/mI
1c~ ~ ~ ~ ~ ~
V(kN)
M(kN'm)0
-62.5
Ascanbeseen,themaximumvalueofverticalshearis
25kNatsupportC.
TheanswerisC.
PPI・ 附W押i2p日sS.com
STRUCTURALDESIGN
Problems46and47arebasedonthefollowinginforma-
tionandillustration.
Thecrosssectionofareinforcedconcretebeamwith
tensionreinforcementisshown.Assumethatthebeam
isunderreinforced.
五=3000lbfjin
2
ん=40,000lbfjin
2
As=3in2
[threeno.9bars]
20in
23in
Problem46
InaccordancewithAmericanConcreteInstitute(ACI)
凶rengthdesign,theallowablemomentcapaciもyof七he
beamismostnearly
(A) 160ft-kips
(B) 180ιkips
(C) 200ft-kips
(D)210ft-kips
Solution
一n
2
i
一如
-
{
3
一
m
一
q
G
一
1
i
ニ
お
ーー
ん一
M
ω
一一一一
ρ'
Inanactualdesignandanalysissituation, 乱 check
shouldalways'bemadetoseeth乱,ttheactualreinforc-
ingsteelratiofallsbetweentheallowablemaximumand
allowableminimumsteelratios,eventhoughthischeck
isnotrequiredtosolvethisspecificproblem.
46 CivilDiscipline-SpecificReviewfortheFE/EITExam
Theminimumrequiredlengthofspiraltransitionbe-
tweenthecurveandroadismostnearly
(A) 28ft
(B) 36此
(0)44氏
(D)72ft
36.Thedesignrequirementsforasectionofhighway
witha1.5%gradeareasfollows.
designspeed=80kmjh
coefficientoffriction=0.35
driverreactiontime= 2.0s
drivereyeheight=1.2m
object(tobe師 oided)height= 0.2m
Thedownhi1ldesignbrakingdistanceforthishighway
ismostnearly
(A)45m
(B) 75m
(0) 100m
(D) 120m
37.Acrestonasectionofhighwayconsistsofavertical
curvewitha1500mradiusandaposi七ive1%gradefol-
lowedbyanegative3%grade.Thedesignrequirements
areasfollows.
designspeed=80kmjh
drivereyeheight= 1.2m
object(tobeavoided)height=0.2m
日七oppingsightdistanceデ300m
Theminimumrequiredlengthofverticalcurveneeded
tosa七isfythedesignstoppingsightdis七anceismost
nearly
(A) 680m
(B) 700m
(0) 760m
(D) 840m
38.A superpavedesignmixtureforahighwaywith
ESALs<107
h回 anominalmaximumaggregatesize
of19mm. Themixturehasbeentestedandhasthe
followingcharacteristics:
airvoids=4.0%
VMAニ 13.2%
VFA= 70%
dust-to-asphaltratio= 0.97
atN = 8gyrationsう Gm mニ 87.1%
atN =174gyrationsぅ Gm m=97.5%
PPI・ 附w.ppi2pass.com
Dothesecharacteristicssatisfy七heircorrespondingsu-
perpaverequirements?
(A) Yes,allthep釘 ametersarewithin組 acc叩七回
ablerange.
(B) No,theVMAisexcessive
(0) No,thedust-to田 asphaltratioistoohigh
(D) No,Gm matNm日 istoohigh.
39.A roadleadingtoas七onequarryistraveledby
40trucks,wi七heachtruckmakinganaverageof10trips
perday. Whenfullyloaded,eachtruckconsis七sofa
frontsingleaxletransmittingaforceof10,000lbfand
tworeartandemaxlesう eachaxletransmittingaforce
of20,000lbf.Theloadequivalencyfactorforthefront
singleaxleis0.0877. Theloadequivalencyf配 torfor
eachreartandemaxleis0.1206.
The18,000lbfequivalentsingleaxleload(ESAL)for
thetrucktra伍conthisroadfor5yrismostnearly
(A) 0.33ESAL
(B) 130ESAL
(0) 48,000ESAL
(D) 240,000ESAL
Problems40and41町 ebasedonthefollowingi1lus七ra-
tion.
↓ド15kN i「↓51NK
FJT:;矛;;「
40.Themagnitudeof七hemaximumbendingmoment
inthebeamismostnearly
(A)6.3kN-m
(B) 14凶 m
(0) 25kN.m
(D)63kN-m
41.Ifthebeamismadeentirelyofsteelandthewhole
beamhasamomentofinertiaabouttheaxisofbending
of2.0X 108
mm4, themagnitudeoftheverticaldefl_ec-
tionatpointDismostnearly
(A) 0.20mm
(B) 2.3m m
(0) 23m m
(D)50mm
Practice Exam I
Prlldiceb:llm1 47
Problems42and43arebasedonthefollowinginforma-
tionandillustra七ion.
Atruckisfacinginitsintendeddirectionoftravelalong
thebeamasshown.
トx
下予 lfm 手ょ :iD
42.Foratrucktravelinginthedirectionshown,七he
ma.ximumverもicalliveloadshearatsuppor七Cismost
nearly
(A)27kN
(B) 76kN
(C) 100kN
(D) 110凶
43.Foratrucktravelinginthedirectionshownう the
ma.ximumliveloadbendingmoment抗 supportCis
mostne紅 ly
(A)80kN-m
(B) 90kN.m
(C) 140kN.m
(D)360kN.m
Problems44and45arebasedDnthefollowinginforma-
tionandillustration.
Aplanetrussspanisshown.Theroadwaybehavesas
simplysupportedbeamspansbetweenthesupporting
lowerchordtrussjoints.Byconvention,positiveforces
aretensileforcesう andnegativeforcesarecompressive
forces.
mA
﹃
I
a
T
i
l
l
-
-
4
7「F
5m 5m 5m
44.Inthex-directionasshown,thema.ximuminflu-
encelineordinatefortensileforceinmemberBFismost
nearly
(A) 0.18kN/kN
(B) 0.36kN/kN
(C) 0.53kN/kN
(D) 0.71kN/kN
45.Inthex-directionasshown,thema.ximuminflu-
encelineordinateforcompressiveforceinmemberBF
ismostnearly
(A) -0.71悶 /kN
(B) -0.53kN/凶
(C) -0却 kN/kN
(D) -0.18kN/kN
Problems46and47arebasedonthefollowinginforma-
tionandillustra七ion.
Thecrosssectionsoftwoshort,concentricallyloaded
reinforcedconcretecolumnsareshown.
だ=4000Ibf/in2
fy= 60,000Ibf/in2
18in 18in
longitundinal
reinforcement
roundspiralcolumn
(crosssection)
(Prob圃 46)
squaretiedcolumn
(crosssection)
(Prob.47)
46.Fortheshortroundspiralcolumn,theapplieda.xial
deadloadis150kips,andtheapplieda.xialliveloadis
350kips. Assumingthatthelongitudinalreinforcing
barsareallthesamesize,theminimumrequiredsizeof
eachlongitudinalreinforcingbaris
(A) no.7
(B) no.8
(C) no.9
(D)no.10
47.Fortheshortsquaretiedcolumn,theapplieda.xial
deadloadis150kips,andtheapplieda.xialliveload
is250kips.Assumingtha七七helongitudinalreinforcing
PPI.附 w.ppi2pass.com
Themomentfunctionsinthedirectionsindicatedby
the1oca1x-coordinateforeachbeamsegmentunderthe
乱ctua110adingareshownonthefollowingmomentdia同
41.Usetheprincip1eofvirtua1workto五nd七hevertica1
de丑ectionatpointD.
1S
40.Constructshearandmome凶 diagrams.Thechange
inshearistheareaundertheapplied1oading,andthe
changeinmome凶 istheareaunder七hesheardiagram.
N│一:-1
札 rニ 州
江lOment
(240,000ESAL)
ムD =~(Jm(自白)
-62.5
The 1argest magnitude of bending
-62.5kN.m(63kN-m)atsupportC.
=240う097ESAL
For5yr,thetota1ESALis
M
川I
V
A
w
h
u
ν
I
E
q
4
J
A
-
-q
川一
y
ベー一
A
h
一
一
円
円
ト
L
﹄
且
晶
mill--
A
TheanswerisD.
TheanswerisD.
M{kN.m) 0
gram.
V{kN)
39.Thetota1ESALpertruckforeachtripis
ESAL七r叫 =(1sing1e砿 1e)(0.0877)
+(2tandemax1es)(0.1206)
=0.3289ESALjtruck-trip
Thetota1dailyESALfor40trucks,eachmaking
10tripsaday,is
Allparametersinthismixturearewithinsuperpave
specifica七ions.
Second,wherestoppingsigh七distanceisgreaterthan
thecurve1eng七h:
(200%)(VHi+♂'2/
L=2S A
Since也eprevioustwoequationsshowthatthestop-
pingsightdistanceis1essthanthecurve1ength,the
minimumrequiredvertica1curve1engthis
L=756.4m (760m)
38.Thismixturewou1dbedesignatedasa19m msu伺
perpavemixture.Thelimitsforsuchamixtureare
airvoids=4.0%
minimumVMλ=13%
VFA=65-75%
dusιto-aspha1tratio=0.6-1.2
atNinit=8gyratiorおう maximumGm m=89%
atNmax=174gyrations,maximumGm m=98%
First,wheres七oppingsigh七distance,8,is1essthanthe
curve1engthう L:
(200%)(Vl五五+VO万五:)2
=(2)(300m)一
1%-(-3%)
L= A8
2
-
(100%)(VWi+伊豆;y
(1%一 (-3%))(300m)2
一 (100%)( 何 日 早 川 南 可f
CivilDisdpline-SpecificReviewfortheFE/日TEx田町1
E叫 = 山cks)(10寄)
X(0印刷)truckωtrip
=131.56ESALjday
3'7.Therearetwoequa七ionstocheck.
TheanswerisA.
TheanswerisC.
=756.4m
PPI.州 w.ppi2pass.com
= 481.0m
56
Solution Practice Exam I
X X
M(kN.m} 0
M =6.25-
Themomentfunctionsin七hedirectionsindicatedbythe
localx-coordinateforeachbeamsegmentunderthevir-
tualunitloadingatpointDareshownonthefollowing
momen七diagram.
Atablesummarizesthemomentfunctionsasfollows.
reglOn regron reglOn
A-B B同 C C自 D
function 0<x<5 0<x<5 0<x<5
M 1.25x 6.25-13.75x -2.5x2
m -0.5x -2.5-0.5x -x
EI EI EI EI
Pr目cticeExam1Solufions 57
ThemodulusofelasticityofsteelisE=2.1X 1011Pa.
Fromtheprincipleofvirtualwork,theverticaldefl.ec-
tionatpoin七Dis
ムD =15
(- EI
+J EI
イー云 2
呼判:+一山1:
I
EI 十半1:十平1:
十年1:
-26.042kN2.m3-78.125kN2.m3
十 390.625 日~2.m3
+286.458kN2.m3
十390.625kN2.m3
(2.1X 10
11
Pa)(品)
/ 紅l立1¥
XIllJlJU-一一ー l
¥ m /
=22.9mm (23mm)
Sincetheunitloadinthevirtualforcesystemwasdown-
wardandtheanswerispositiveinsign,theactualde-
fl.ectionisalsodownward.
TheanswerisC.
PPI.附 w.ppi2poss.com
58 CivilDiscipline-SpecificReviewfortheFEjEITExam
42.Constructanduseanin丑uencelineforverもicalshear
atsupportC.Thein乱uencelineisconstructedbyplot-
tingthechangeinresponseonafree四 bodydiagramof
asectionofbeamatsuppo吋 Casaunitloadtravels
acrossthestructure.
。
745m 5m
influencelineforVc(kN/kN)
loadposition
formaximum
verticalshear
1.5
t t71.17kN 17.79kN
1 _ 1
Theloadpositionshownresultsin出 emaximumre司
sponseinthebeamatsuppor七Cfor出especi五eddi-
rectionoftravel. Themaximumverticalshearcanbe
foundbysuperposition回
日 x =(1.5~~) (71.17凶)
Thisproblemaskedforthemagnitudeofmaximumver-
ticalshearatsupportCforatrucktnαvelinginthe
directionshown.Ifthisproblemhadaskedforthemag-
nitudeofmaximumverticalshearatsupportCforthe
giventruckαxleconfigur,αtion,theaxleloadposi七ions
wouldhavetobeswitchedaroundandbothaxleloads
placedontheinfiuencelinewiththeheavieraxleload
onthelargerinfiuencelineordinateandthelighteraxle
loadonthesmallerin丑uencelineordinate.Thiswould
resultinalargernumericalanswer.
TheanswerisD.
43.Constructanduseaninfiuencelineforbendingmo-
ment抗日uppo凶 C.Thein丑uencelineisconstructedby
plottingthechangeinr田 ponseona丘ee四 bodydiagram
ofasectionofbeamatsupportCasaunitloadtravels
across七hestructure.
PPI.附 w.ppi2p日sS.com
。
loadposition
formaximum
bendingmoment
t t
Theloadpositionshowngivesthemaximumresponse
inthebeamatsuppor七Cforthespeci五eddirec七ionof
travel. Themaximumbendingmomentcanbefound
bysuperposition出
時 max= 1(-5守)川
= 355.8kN-m
Thisproblemaskedforthemagnitudeofmaximum
bendingmomentatsupportCforatrucktravelingin
thedirectionshoωn.Ifthisproblemhadaskedforthe
magnitudeofmaximumbendingmomentatsupportC
forth巴 giventruckαxleconfigurαtionう theaxleloadpo-
sitionswouldhavetobeswitchedaroundandbothaxle
loadsplacedontheinfiuencelinewiththeheavieraxle
loadonthelargerin宜uencelineordinateandthelighter
axleloadonthesmallerin丑uencelineordinate.This
wouldresultinalargernumericalanswer.
TheanswerisD.
44.Constructaninfiuencelinefortheforceinmem-
berBF.Thein丑uencelineisconstructedbyplotting
thechangeinresponseinmemberBFasaunitload
travelsacrossthestructure.
_J.,"
ゆ E ; f d務
lllJzrJ iiλ哩~I(附/kJ
Whenmovingtheuni七loadacrossatrussstructure,this
loadmustbedisむibutedtothetwojoints.Itisstated
七hattheroadwayactsassimplebeamspansbetween
trussjoin七s.Therefore,whenaloadisplacedbetween
trussjoin七s,thetwojointsadjoiningthebeamspanact
assimplebeamsupportsandthemagni加 desofthe
loadsappliedto出etwotrussjointsarethesameasfor
thecalculatedreactionforcesof七hisbeam.
ThemaximumordinatefortensileforceinmemberBF
is0.53kN/kN
TheanswerisC.
45.Fromtheinfl.uencelinein801.44,themaximum
ordinateforcompressiveforceinmemberBFis-0.53
kN/kN
TheanswerIsB.
46.Determinetheamountofreinforcings七eelrequired
bytheminimumrequiredreinforcementratio,Pg,of
0.01.Theminimumareaofreinforcings七eelrequiredis
=(0川(年)
Determinetherequiredamountofreinforcingsteelbased
onthefactoredaxialload,Pu.
Puニ1.2Pdead+1.6P!ive
ヱ(1.2)(150kips)+(1.6)(350kips)
= 740kips
Thenominalaxialcompressiveloadcapacityisgivenby
凡 =0.85P.。
= (0.85)(0.85f~Aconcrete +んAs)
= (0部 )(0.85五(Ag-As)+んAs)
Pr目di(eExam1Solutiolls S9
I七isrequiredthatゆPn2:Pu. Foraxialcompression
withspiralreinforcement,ゆ=0.70.8ettingゆPn=P,叫
andsolvingfortheareaoflongitudinalreinforcingsteel
gIVes
山 ps)(10002)
485)(4000主)(年)
。主一(0.85)(4000~~;)
As= 6.69in2,requiredforthegivenappliedaxialcom-
pressiveloads,isgreaterthanAs= 2.54in2basedon
theminimumallowedreinforcementratio,Pg=0.01.
TheminimumrequiredareaofreinforcementisAs二
6司 69in2.
Thecolumnhassixlongitudinalreinforcingbars.The
requiredareaofeachlongi七udinalreinforcingbaris
A=~ 一日主主一 -nbars 6bars
= 1.11in2
/bar
A barareaof1.11in2issatisfiedbyano.10bar,
whichhasanominalareaof1.27in2.
Inanactualdesign/analysissituation,乱 checkshould
alsobemadetoseethattheactuallongitudinalrein-
forcementratiodoesnotexceedthemaximumallowable
ratioof0.08.
TheanswerisD.
47.Determinetheamountofreinforcingsteelrequired
bytheminimumrequiredreinforcementratio,内, of
0.01.Theminimumareaofreinforcingsteelrequiredis
As= pgAg
= (0.01)(18in)2
=3.24in2
Determinetherequiredamoun七ofreinforcingsteelbased
onthefactoredaxi,alloadう Pu.
Pu=1.2Pde吋+1.6P!ive
=(1.2)(150kips)+(1.6)(250kips)
=580kips
PPI.附 w.ppi2p日sS.com
70 CivilDiscipli鵬闘SpedficReviewfortheFE/EITEx脚
The七ra伍cflowrelationshipisgivenbyq=kv,inwhich
qisthetra伍cvolumeinveh/hr.Themaximumtra伍c
volumefor七hisroadismostnearly
(A) 760veh/hr
(B) 880veh/hr
(C) 900veh/hr
(D)960veh/hr
38.Thes七oppingsightdis七anceis430ftforadesign
speedof50mphonasectionofhighway. Thegrades
forthishighwaysectionare-1%followedby3%.The
requiredlengthofverticalcurveneededtosatisfy七he
AASHTOstoppingsightdistanceforthisdesignspeed
ismostnearly
(A) 270氏
(B) 380氏
(C)410ft
(D)450ft
39.Aone-laneruralroadhasa100
curveextendingfor
230m alongitscenterline.Theroadis5mwidewith
3m wideshoulders.Thedesignspeedforthisroadis
75km/h.
Thesuperelevationneededsothatsidefrictionisnot
neededismostnearly
(A) 0.00050
(B) 0.034
(C) 1.1
(D) 1.9
40.Thewornsurfacecourseofahigh-volumepave四
mentisbeingreplacedwithadesignrequiringatotal
structuralnumberof6.6.Theengineerhasdecidedto
replace6inofthesurfacewithrecycled-in-placeasphalt
concretehavingasurfacecoursestrengthcoe血cientof
0.42,leavinginplace3inofsoundoriginalpavement
havingastrengthcoefficientof0.3. Undertheorigi-
nalpavementarea10inceme凶 -treatedbasehaving
astrengthcoe血cientof0.20ぅ and阻 8insandygravel
subbase.Whatistheminimumstrengthcoefficientfor
thesubbase?
(A) 0.05
(B) 0.10
(C) 0.15
(D)0.20
PPI. 州w.ppi2p日sS.com
Problems41and42arebasedonthefollowinginforma-
tionandillustration.
Thebeamshownisloadedwithtwo1000Npointloads.
Theseparationismaintained抗 2m,but出eloadsmay
bemovedtoanylocationonthebe乱m.
h
門
⋮
什
﹁
一
助
│
蕩Z,
10m 10m
41.ThemaximumvalueforshearatsupportAismost
nearly
(A) 2000N
(B) 2800N
(C) 3000N
(D)3800N
42.Themaximumvalueformomentatsuppor七A is
mostnearly
(A) 1.8kN.m
(B) 8.0kN-m
(C) 10kN.m
(D)18kN-m
43.Atriangularpin-connectedtrusscarriesaloadof
4448Nasshown.Eachmemberhasthesamemodulus
ofelasticityandcross-sectionalarea.
:%:l M
3.0m
P
N
4448N
4.6m
Practice Exam II
ThetrussmemberpropertiesareE=200X 106
kPa
andAニ 2580.6mm2.Theverticaldeflectionatpoint
Pismostnearly
(A)0.25mm
(B) 0.48mm
(C) 0.51mm
(D)0.75mm
Problems44-46arebasedonthefollowingillustration.
l~ ~ ~ ~ ~ ~ ~ ~ ~ ~ ~ ~ ~ ~ L~
A 易 易
3 m 3 m
44.IfthereactionatsupportAis18.75N,thereaction
ateachof出eoutersupportsismostnearly
(A) 5.6N
(B) 7.2N
(C) 11N
(D)14N
45.Themaximumvalueofverticalshearatanypoint
alongthebeamismos七nearly
(A)4.7N
(B) 9.4N
(C) 14N
(D) 18N
46.Themaximumvalueofmomentatanypointalong
thebeamismos七nearly
(A) 1.2N.m
(B) 3.1N.m
(C) 4.6N.m
(D)5.7N.m
Pr目diceEx園田12 71
Problems47-49arebasedonthefollowinginformation
andillustration.
Thespanlengthandcrosssectionofareinforcedcon-
cretebeamareshown. Thebeamisunderreinforced.
Theconcreteandreinforcingsteelpropertiesぽ e五=
3000lbf/in
2
うん =40,000lbf/in
2
,andAs=3in2
•
6ft
10ft
18i日目
beam
cross
section
47.Neglec七ingbeamself-weightandbasedonlyonthe
allowablemomentcapacityofthebeamasdetermined
usingAmericanConcreteInstitute(ACI)strengthde白
signspecificationsう themaximumallowableliveloadis
mostnearly
(A)23,000lbf
(B) 29,000脳
(C) 35,000lbf
(D)50,000lbf
48.Thebeamsupportsaconcentraもedliveloadof
50,000lbf.Neglec七 beamself-weight.Theminimum
amoun七 ofshearreinforcementrequiredforacenter-
to-centerstirrupspacingof12inunderACIstrength
designspecificationsismostnearly
(A)0.18in2
(B) 0.36in2
(C) 0.67in2
(D)0.78in2
49.Thebalancedreinforcingsteelratioforthisbeam
inaccordancewithACIspecificationsismostnearly
(A) 0.037
(B) 0.043
(C) 0.051
(D) 0.058
PPI.附 w.ppi2pass.com
Thedensityis33.125veh/miformaximumtra缶cvol司
ume.Substitutionin七othetra白cflowrelationshipgives
themaximumtra血cvolume.
= (53::)(33.125:~)
一 (0.8品)(33.1叫
TheanswerisB.
38.Sincethereisanegativegradeprecedingapositive
gradeう thisisasagverticalcurve.Usingthesagverti-
calcurveequationsfromthecivilengineeringsectionof
theNCEESHandbook,thealgebraicdifferencebetween
gradesis
A=1-1% 3%1
=4%
Wherethestoppingsightdistance,8,isless七han七he
verticalcurvelength,Lう
L=-竺L-
400十 3.58
(4)(430ft)2
400十 (3.5)(430此)
=388.2ft
Wherethestoppingsightdistanceisgreaterthanthe
curveleng七h,
400+3.58
L二 28一 一 A一 一
400+(3.5)(430氏)
=(2)(430此) - 4
=383.8ft
Fromthe七wovaluesforlengthofcurveう itcanbeseen
thatthes七oppingsightdistanceisgreaterthan七he
curvelength. Therefore,therequiredverticallength
ofcurveis
L=383.8抗 (380此)
TheanswerisB.
39.Theradiusofcurvatureis
R=~= 川寺)
PradiceEx日m2501日tiol1S 81
Foraside-台ictionfactoroff=0,thesuperelevationis
givenby
((75引(1吋(品))
(9.81~) (1318m)
TheanswerisB.
40.Thestructuralnumberisthesumofproductsof
thelayerdepths(thicknesses)andstrengthcoe血cients.
SN=αrecycleDrecycle十 αoriginalsurfaceDoriginalsurface
+αbaseDbase十 αsubbaseDsubbase
6.6=(0.42)(6in)+(0.3)(3in)十 (0.2)(10in)
十αsubbase(8in)
αsubbase=0.15
TheanswerisC.
41.Drawanin丑uencelineforshearatsuppor七Awith
loadspositionedasshownformaximumshearvalue.
Forexampleう forshearatsupportAう usingbasicbeam
statics,
20m
2.0
influencelineforshearatsupportA
ト --1
一一一:51~20loadsplacedformaximumshearvalueatsupportA
ThemaximumvalueforshearatsupportAis
VA=(1000N)(1.8)+(1000N)(2.0)
=3800N
TheanswerisD.
PPI.剛 w.ppi2p日sS.com
Solution Practice Exam II
82 CivilDisdplille-SpedficReviewfortheFEjEITEx日m
42.AninfiuencelineforbendingmomentatsupportA
canbedrawnwithloadspositionedasshownformax-
imummome凶 value. Asin五nding七heinfiuenceline
forshearう aunitloadismovedacrossthebeamandthe
variationinbendingmomentatpointAisgraphed.
姦
10m
す
m
A
10m
influencelineformomentatsupportA
loadsplacedformaximumvalueatsupportA
ThemaximumvalueformomentatsupportA (dis-
regardthesignぅ sincethemaximumvalueiswanted)
lS
MA= (1000N)(8.0m)+(1000N)(10.0m)
二 18000N.m (18kN.m)
TheanswerisD.
43.Theprincipleofvirtualworkcanbeusedtofind
thedefiectiona七poin七P.
Choosethepositivedirectionsasupwardandtothe
right.Choosepositivemomentsasclockwise.
LMN=ON
=RM,, (3.0m)十 (4448N)(4.6m)
RM"=6820N [七othe1eft]
エMM=ON
=-RN"(3.0m)+(4448N)(4.6m)
RN"=6820N [totheright]
LFy=0N
=RMy - 4448N
RMν=4448N [upw乱rd]
PPI.附 W押 i2p日sS.com
Byinspection,thex-componentofforceinmemberMP
isthesamemagnitudeandoppositedirectionasthe
reactionatpointM.
N
一
e
U
一
0
0
一
2
M
で
4
一
R
I
一弘
一
十
一
イ
、
一
-
m
一十
一
﹀
d
l
u
一
勺G
広
明
一
向
4
1‘
,
一
円
U
﹁
│
二
一
白
川
M
伝
山
、
必
R
U
v
剖
p
i
M
M
一-6820N
8142N
3.0m ON
P
一一 6820N
N 6820N
4.6m
ApplyingaunitloadatpointPproducesthefollowing
virtualforces.
M
....1.5N
1.8N
3開 O mI 0N
P
1.5N
N 1.5N
4.6m
Thefollowingtablesummarizestheactualandvirtual
forces.
FQ
member (N)
MP 8142
NP 6820
MN 0
Fp
(virtualforce)
L
(m)
FQFpL
(N-m)
1.8
1.5
0
5.49 80459
4.6 47058
3.0 0
total=127517
Calcu1atethevertica1de丑ection抗 pointP.
ムp=LFQ5L=玄FQ22
=占LFQFpL
c川2却別ル一川Oω川 い 川O川 山 川1叩川O併6印刷叫叫)べ中(や1∞
x(2附 mm2
)(抗日
x(127517N-m)(1000空)
TheanswerisA.
44.Fromthe1awsofequi1ibriumう eachreactionatan
outersuppor七is
ニ (t)((5:)州-18同
TheanswerisA.
45.Themaximumshearcanbedeterminedfroma
sheardiagram. (A1thoughno七necessaryforthisprob-
1em,amomentdiagramhasa1sobeenconstructed,and
thisisusedforsolvingthenextprob1em.)
Thechangeinshearistheareaunderthe10addiagram
uptothatpoint.Uptothecenterofthebe叩 1,pointA,
Atpoin七A,
ド 5.6N-(5:)(3m)
V=-9.4N+18.75N
= 9.35N (9.4N)
Themomentisequa1totheareaunderthesheardia-
gram.
Pr自ctic告Ex自m2Soluti開 5 錦
Thechangeinmomenもisgivenbytheareaunder七he
sheardiagramuptothatpoint.
= (t)(5.6叩 2m)
ル 3m =凡=1.12m 十(訪問問問m)
? ? ?
x=1.12m 1.88m 1司 88m
3膚 1 3.1
M(N'm)
5.7
Fromthesheardiagram,themaximumva1ueforshear
occurs剖 supportA.
Vmax= 9.4N
TheanswerisB.
46.Fromthemomentdiagramin801.45,theva1u巴for
maximummomentoccursatsupportA.
Mmax= [-5.7N.m[
= 5.7N-m
TheanswerisD.
PPIe附 w.ppi2p日sS.com
哩50 Appendix AftemoonSa円lpleExamination
42. 1nusingch10rinefordisinfection
.a. concen住ationofHOC1ispH-dependent
b. [HOCl]equals[OCllatpH7
c. e旺'ectivenessincreaseswithtimeofcontact
d. totaleliminationofbacteriaispossible
43. A61・cm-diametercircularconcretesewerissetonaslopeof0.00l.The
flowwhenitishalιfullis
ajof伽 叫 制l
b. 0.08m""/s
c. 0.20m
3
/s
d. 0.28m
3
/s
44. ArigidbarBCDissupportedbyahingedconnectionatsupportBand
byacableACasshown.ThereactionforceatsupportBis
A
D
B
ド
2.0m 、、レO.5m....J
a. 2.50kN
b. 14.20kN
c. 9.70kN
d. 14.42kN
45. AsimplysupportedbeamABissubjectedtoauniformlydistributed
loadandaconcentratedloadasshown.Themidspandef1ectionis
20kN
ド 3.0m
E=200GPa
1=3x106
m m
4
a. 16mm
b. 38mm
c. 22mm
d. 8mm
『
."
3
4
3.5 m
Practice Exam III
..
r--r・一一一一r-
[}=[]
SampleExam 15.
46. Asteelcolumn(W21x93)hasthebottomendsupportedrigidlysoas
topreventbothrotationandtranslation.Thetopofthecolumnisfree
totranslateandrotatein出eplaneofthediagrambutpinned(rotation
allowed,translationprevented)intheperpendiculardirection.The
lengthofthecolumnis32ft.Thecriticalbucklingloadasgivenby
Euler'stheoryis
a. 13.4kips
b. 36.7kips
c. 366kips
d. 1002kips
47. Thethree-hingedarchABCcarriesauniformlydistributedloadfroma
horizontaldeckasshown.百levalueofthehorizontalthrustatthesupportsis
ー
ド 15m 当6 10m
オ
a. 282kN
b. 197kN
c. 218kN
d.252kN
48. Giventhetrussshown,theverticaldeflectionatBis
10抗
ド
8ft
a. 1.2in
b. 0.05in
c. 0.2in
d. 0.1in
当│
B
E=29,000ksiforbothmembers
A=2in2
forAB
A=3in2
forBC
20kips
6 m
4 m
152 AppendixAftemoonSampleExamination
49. Themaximumbendingmomentforthebeamshownis
ド 3.5m
~〈
1.5m
a. 12.60kNm
b. 8.82kNm
c. 9.41kNm
d. 10.25kNm
50. Aclientrequiresanewfaci1ityquicklytoreplaceonedestroyedbya
hurricane.Heisnotcertainexactlywhatitshouldinclude.Thebest
typeofcontracttobeusedin出issituationis
a. unitprice
b. lumpsum
c. costplusfixedfee
d. surety
51. Whatistheheight(白)ofaconicalspoilpile,if100ftbankcuydof
commonearthwithanangleofreposeof320
anda12%swellis
deposited?
a. 9.6
b. 19.2
c. 10.4
d. 33.3
52. Determinethevolumeoffi1lforawholestation(100ft)ofcutwith
theendareasshownbelow.
¥Al=120ft'/¥ι=150が/¥A'=100ft,/
a. 820yd3
b.456.8ydj
c. 506.2yd
J
d.569.4yd
J
53. ABCConstructionCompanyisthelowbidderat$500,000onabuilding
project.Thecompanyhasprovidedthetypicalbidbond.Afteraward
ofthecontract,thecompanydecidestowithdrawitsbid.Thesecond
lowestbidwas$520,000.Howmuchisthesur~ty .company requiredto
reimbursetheclientforthefailureofABCtosign?
a. $500,000
b. $50,000
c. $20,000
d. $5000
Solutions 唱63
41. d. Allofthestatementsareco町 ect.
42. a.
43. b.
Qf=0.2m
3
/s
Q/Qf=0.4 内
Q=0.08m'>/s
44. d. ThefreebodydiagramforBCDisshown.Note白atacommonmistake
couldbe脳 出 国ngthatcableACformsa3-4-5凶anglewhenitd<∞sn't.百le
ve同calandhorizontalcomponentsofT訂 eO.66Tand0.75T, respectively.
T
10
Bx
ヱMB=0.66Tx2-10x2.5=0司 T=ω 4
ヱぇ=久一0.75x18.94=0弓 Bx=14.20
ヱπ =久+0“x18.94-10=0=>By=-2.50
RB=石弓4
45. b. EI=r2xぽ型)X(3X1O-6m4)=6∞kNm2
¥ m-J
Deflectionatrnidspan(x=2m)duetopointloadisgivenby
Pb 三 句 今 20xO.5 __,
o=一一一[-x
j
+(Ll
ーか)x]=_-~:. ~~:~ [-2j
+(4l
- OS)x2]=0.016
6LEr " 6x4xω。
Deflectionatrnidspan(x=2m)duetodistributedloadisgivenby
δ 5wL
4
5x4x4
4
=一一=一一一一 =0.022
384EI 384x600
Totalmidspandeflection=0.038m=38mm
46. c. Bucklingaboutthestrong(x)鉱 is:
K=2.0;L=32ft;rx=8.70in;KUr=88.3
Bucklingabouttheweak(y)axis:
K=0.7;L=32ft;ry=1.84in;KUr=146.1
Solutions may be for problems with slightly different
dimensions;
I was given two apparently disjointed sets.
One may conclude that problems are for the most part
identical but with different dimensions.
唱64 Appendix AftemoonSampleExamination
R π2EA π2x29,000x273 内 〆 " .
= ナ ー ー す = ヲ =jOO阻 p:
(子}
47. b. Therearefourextemalreactions-A_A.c..andC-andtwointemalx' "Y ~x'
hingeforces-BxandBy一 旬 bedeterrnined.RecognizethatAxandCx
areequalandopposite.SolvingforeitherAxorCxissufficient.
TakingmomentsaboutA(entirestructure):
ヱMA =500x山 +4ι -25Cy=0
TakingmomentsaboutB(righthalfofstructure):
ヱMB.right=200x5-6に一以 =0
Solvingtheseequations,wegetCx=ー197.4kips.
48. d. Step1:UsingmethodofjointsatjointB:
凡B =+25.6k(T)
FBC=ー16.0k(c)
Step2:Calculatethememberforcesduetovirtualload(note:in出iscase,
出e毘 alloadandthevirtualloadlooksimilar,sowecanusescaling):
Step3:
んB=+1.28(T)
fBC=-0.80(c)
B
=す rFfL1=~5似1.28 x(12.81x12)+-16x-O.8x(8xI2L=0.10血
""""'lAE) 2x29.000 3x29.000
49. b. Usingthefixedendmomentloadcasesin白eFESupplied-Rそference
Handbookwiththefollowingdata:
P=12kN;α=3.5m;b=1.5m;L=5m
....
=-.:
Solutions 唱65
Pab2
12x3.5x1.52
=ーで一 =--_._.__--_.- =3.78kNrn
AB 72
L~
Pa2
b 12x3.52
x1.5=ーで一=--_._.__-._.-==8.82kNrn
BA r2L~
12
TakingrnornentsaboutA,
ヱMA =+3.78一 以 3.5-8.82+玖 =0叫 =9.408釦 dAy=2.592
Thesheardiagrarnandbendingrnornentdiagrarns紅 e:
2.592
9.408
+5.292kNm
-3.78kNm -8.82kNm
50. c. Costp1usfixedfee.Choicesaandbrequireadetaileddesign;choiced
isabondingcornpany.
51. c. Loosecuft=100cuydx1.12x27cuft/cuyd=3024cuft
Basediarneter(B)==(7.64x3024/tan320
)1/3=33.3ft
Height==(33.3/2)(tan320
) ==10.4ft
52. d. 百1isproblerncanbesolvedusingtheprisrnoidalrnethod.Vo1urne==(1
cuyd/27cuft)1∞ft[120sqft+(4x 150sqft)+100sqft]/6=
506.2cuyd
53. c. Theclientcanonlyrecoverthe10ssincurredbythedefault,whichis
出edi百erencebetweenthelowbidandthesecond10westbid:$20,000.
54. b. Totalcost=$50,000+$5000==$55,000
Unitcost==$55,000/200==$275.00/ftNote出attheunitpricernust
includethecontractor'sprofit/rnarkup.
Determinacy and Stability
Determinacy and Stability I
Trusses are statically determinate when all the bar forces can be determined
from the equations of statics alone. Otherwise the truss is statically
indeterminate.
A truss may be statically/externally determinate or indeterminate with respect to
the reactions (more than 3 or 6 reactions in 2D or 3D problems respectively).
A truss may be internally determinate or indeterminate.
If we refer to j as the number of joints, R the number of reactions and m the
number of members, then we would have a total of m + R unknowns and 2j (or
3j) equations of statics (2D or 3D at each joint). If we do not have enough
equations of statics then the problem is indeterminate, if we have too many
equations then the truss is unstable.
2D 3D
Static Indeterminacy
External R > 3 R > 6
Internal m + R > 2j m + R > 3j
Unstable m + R < 2j m + R < 3j
Victor E. Saouma; CVEN 3525; Univ. of Colorado Reactions 5/23
TRUSSES
Method of Joints
Method of Joints II
This method should be used when all member forces must be determined.
In truss analysis, there is no sign convention. A member is assumed to be
under tension (or compression). If after analysis, the force is found to be
negative, then this would imply that the wrong assumption was made, and that
the member should have been under compression (or tension).
On a free body diagram, the internal forces are represented by arrow acting on
the joints and not as end forces on the element itself. That is for tension, the
arrow is pointing away from the joint, and for compression toward the joint.
cc
t
C
A B
-ve
+ve
-ve
A B
C
Victor E. Saouma; CVEN 3525; Univ. of Colorado Reactions 9/23
Example; Method of Joints
Example; Method of Joints I
A
B C D
E
F
G
H
12
8
20 k 40 k 40 k
10'
32'
24' 24' 24' 24'
1 R = 3, m = 13, 2j = 16, and m + R = 2j
√
2 We compute the reactions
+
 
¡
 ΣME
z = 0; ⇒ (20 + 12)(3)(24) + (40 + 8)(2)(24) + (40)(24) − RAy (4)(24) = 0
⇒ RAy = 58 k 6
+ ?ΣFy = 0; ⇒ 20 + 12 + 40 + 8 + 40 − 58 − REy = 0
⇒ REy = 62 k 6
Victor E. Saouma; CVEN 3525; Univ. of Colorado Reactions 10/23
Example; Method of Joints
Example; Method of Joints II
3 Consider each joint separately:
Node A: Clearly AH is under compression, and AB under tension.
58 k
FAB
FAH
A
+ 6ΣFy = 0; ⇒ −FAHy + 58 = 0
FAH = l
ly
(FAHy )
ly = 32; l = 322 + 242 = 40
⇒ FAH = 40
32
(58) = 72.5 k Compression
+- ΣFx = 0; ⇒ −FAHx + FAB = 0
FAB = Lx
Ly
(FAHy ) = 24
32
(58) = 43.5 k Tension
Node B:
Victor E. Saouma; CVEN 3525; Univ. of Colorado Reactions 11/23
Example; Method of Joints
Example; Method of Joints III
B
20 k
FBH
43.5 k FBC
+- ΣFx = 0; ⇒ FBC = 43.5 k Tension
+ 6ΣFy = 0; ⇒ FBH = 20 k Tension
Node H:
H
12 k
FHG
FHC
72 k
20 k
FHCx
FHCy
FAHy
FHGx
FHGy
FAHx
Victor E. Saouma; CVEN 3525; Univ. of Colorado Reactions 12/23
Example; Method of Joints
Example; Method of Joints IV
+- ΣFx = 0; ⇒ FAHx − FHCx
+ FHGx
= 0
43.5 − 24
242+322
(FHC ) + 24
242+102
(FHG) = 0
+ 6ΣFy = 0; ⇒ FAHy + FHCy
− 12 + FHGy
− 20 = 0
58 + 32
242+322
(FHC ) − 12 + 10
242+102
(FHG) − 20 = 0
This can be most conveniently written as
0.6 −0.921
−0.8 −0.385
FHC
FHG
=
43.5
26.0
Solving we obtain FHC = −7.5 and FHG = −52, thus we made an erroneous assumption in the
free body diagram of node H, and the final answer is
FHC = 7.5 k Tension
FHG = 52 k Compression
Node E:
Victor E. Saouma; CVEN 3525; Univ. of Colorado Reactions 13/23
Example; Method of Joints
Example; Method of Joints V
E
62 k
FED
FEF
ΣFy = 0; ⇒ FEFy = 62 ⇒ FEF =
242+322
32
(62) = 77.5 k C
ΣFx = 0; ⇒ FED = FEFx ⇒ FED = 24
32
(FEFy ) = 24
32
(62) = 46.5 k T
The results of this analysis are summarized below
Victor E. Saouma; CVEN 3525; Univ. of Colorado Reactions 14/23
Example; Method of Joints
Example; Method of Joints VI
43.5 43.5 46.5 46.5
72.5
20
32
2.5
7.5
4
0
77.5
46.5
6258
43.5
52 52
58 62
12
8
20 40 40
4 We could check our calculations by verifying equilibrium of forces at a node not previously used, such as D
Victor E. Saouma; CVEN 3525; Univ. of Colorado Reactions 15/23
Examples Beams
Example Beam; 1
A
B C D
E
2 k/ft
4ft 4ft 4ft6ft
4
3
11 k 10 k
Reactions are determined from the equilibrium equations
(+ ) ΣFx = 0; ⇒ −Ax + 6 = 0 ⇒ Ax = 6 k
(+  ¡) ΣMA = 0; ⇒ (11)(4) + (8)(10) + (4)(2)(14 + 2) − Ey (18) = 0
⇒ REy
= 14 k
(+ 6) ΣFy = 0; ⇒ Ay − 11 − 8 − (4)(2) + 14 = 0 ⇒ Ay = 13 k
Victor E. Saouma; CVEN 3525; Univ. of Colorado Internal Forces 11/31
SHEAR AND MOMENT DIAGRAMS
Examples Beams
Example Beam; 2
A
B C D
E
2 k/ft
4ft 4ft 4ft6ft
4
3
11 k 10 k
Shear are determined next.
1 At A the shear is equal to the reaction and is positive.
2 At B the shear drops (negative load) by 11 k to 2 k.
3 At C it drops again by 8 k to −6 k.
4 It stays constant up to D and then it decreases (constant negative slope
since the load is uniform and negative) by 2 k per linear foot up to −14 k.
5 As a check, −14 k is also the reaction previously determined at F.
Moment is determined last:
1 The moment at A is zero (hinge support).
2 The change in moment between A and B is equal to the area under the
corresponding shear diagram, or ∆MB−A = (13)(4) = 52.
3 etc...
Victor E. Saouma; CVEN 3525; Univ. of Colorado Internal Forces 12/31
Examples Beams
Example Beam; 3
A B C D E
2 k
-6 k
-14 k
Slope= dV/dx=w=-2
dM/dx=+2 dM/dx=-6
0+52=52
52+12=64
64-24=40
10 k
11 k
2 k13 k
8 k
2 k
A
B
C
E
13 k
A=(13)(4)=52 A=(6)(2)=12
A=(-6)(4)=-24 A=-4(6+14)/2=-40
-6 k
Victor E. Saouma; CVEN 3525; Univ. of Colorado Internal Forces 13/31
arches Three Hinged Arch; Point Loads
Three Hinged Arch; Point Loads I
80' 60'
33.75'
30'
20 k
B
80 k
26.25'
A
C
20' 20'
20 kB
30 k
80 k
A
C
HA
VA
VC
HC
VB VB
HB HB
30 k
Four unknowns, three equations of equilibrium, one equation of condition ⇒ statically determinate.
+
 
¡ ΣMC
z = 0; (RAy )(140) + (80)(3.75) − (30)(80) − (20)(40) + RAx (26.25) = 0
⇒ 140RAy + 26.25RAx = 2.900
+
- ΣFx = 0; 80 − RAx − RCx = 0
+ 6ΣFy = 0; RAy + RCy − 30 − 20 = 0
+
 ¡ ΣMB
z = 0; (Rax )(60) − (80)(30) − (30)(20) + (RAy )(80) = 0
⇒ 80RAy + 60RAx = 3, 000
(22)
Victor E. Saouma; CVEN 3525; Univ. of Colorado Cables  Arches 19/24
ARCHES
arches Three Hinged Arch; Point Loads
Three Hinged Arch; Point Loads II
Solving those four equations simultaneously we have:




140 26.25 0 0
0 1 0 1
1 0 1 0
80 60 0 0







RAy
RAx
RCy
RCx



=



2, 900
80
50
3, 000



⇒



RAy
RAx
RCy
RCx



=



15.1 k
29.8 k
34.9 k
50.2 k



(23)
We can check our results by considering the summation with respect to B from the right:
+
 
¡

ΣM
B
z = 0; −(20)(20) − (50.2)(33.75) + (34.9)(60) = 0
√
(24)
Victor E. Saouma; CVEN 3525; Univ. of Colorado Cables  Arches 20/24
Examples Beam
Example: Beam I
Determine the deflection at point C. E = 29, 000 ksi, I = 100 in4
.
2 k/ft
A
A B
B
C C
15 k 45 k
0.5 k 1.5 k
1 k
20' 10'
C
-x-0.5x
15x-x2
-x2
Real Moment Virtual Moment
x x
Element x = 0 M δM
AB A 15x − x2 −0.5x
BC C −x2 −x
Victor E. Saouma; CVEN 3525; Univ. of Colorado Virtual Work 10/23
BEAM DEFLECTION, VIRTUAL FORCE
Examples Beam
Example: Beam II
Applying the principle of virtual work, we obtain
∆CδP
δW
∗
=
L
0
δM(x)
M(x)
EIz
dx
δU
∗
(1)∆C =
20
0
(−0.5x)
(15x − x2)
EI
dx +
10
0
(−x)
−x2
EI
dx
=
2, 500
EI
∆C =
(2, 500) k − ft3
(1, 728) in3
/ ft3
(29, 000) ksi(100) in4
= 1.49 in
Victor E. Saouma; CVEN 3525; Univ. of Colorado Virtual Work 11/23
Examples Truss; Simple
Truss; Simple I
Determine the deflection at node 2 for the truss.
12'
1
1
3
4
4
7
5
2
3
5
60 k 120 k
6
12' 12'
A=5.0 in2
each;
E=10x103
ksi 60 120
-117.3-83.8
37.5 52.5
16.8
-16.8
-45.0
75.0 105.0
A
-0.50
-0.56 -0.56
0.25 0.25
0.56
0.56
0.5 0.51.0
-0.54
-1.124
-
1.574
0.45 0.63
0.226
-0.226
δP
(e)
P(e), L, A, E, δP
(e) P(e)
L
AE
Member kips kips ft in2 ksi
1 +0.25 +37.5 12 5.0 10 × 103 +22.5 × 10−4
2 +0.25 +52.5 12 5.0 10 × 103 +31.5 × 10−4
3 -0.56 -83.8 13.42 5.0 10 × 103 +125.9 × 10−4
4 +0.56 +16.8 13.42 5.0 10 × 103 +25.3 × 10−4
5 +0.56 -16.8 13.42 5.0 10 × 103 −25.3 × 10−4
6 -0.56 -117.3 13.42 5.0 10 × 103 +176.6 × 10−4
7 -0.50 -45.0 12 5.0 10 × 103 +54.0 × 10−4
+410.5 × 10−4
Victor E. Saouma; CVEN 3525; Univ. of Colorado Virtual Work 18/23
You will not get something as complex as this truss
TRUSS-DEFLECTION; Virtual
force
Examples Truss; Simple
Truss; Simple II
The deflection is thus given by
δP∆ =
7
1
δP
(e) PL
AE
∆ = (410.5 × 10−4
)(12 in/ ft) = 0.493 in
Victor E. Saouma; CVEN 3525; Univ. of Colorado Virtual Work 19/23
Examples Truss with initial camber
Truss with initial camber I
It is desired to provide 3 in. of camber at the center of the truss shown below
1
32
6 @ 27'
1 k
36'
by fabricating the endposts and top chord members additionally long. How much should the
length of each endpost and each panel of the top chord be increased?
Assume that each endpost and each section of top chord is increased 0.1 in.
Member δP
(e)
int ∆L δP
(e)
int ∆L
1 +0.625 +0.1 +0.0625
2 +0.750 +0.1 +0.0750
3 +1.125 +0.1 +0.1125
+0.2500
Victor E. Saouma; CVEN 3525; Univ. of Colorado Virtual Work 22/23
You may
get
somethin
g along
those
lines, or
with
temperat
ure load
Examples Truss with initial camber
Truss with initial camber II
Thus,
(2)(0.250) = 0.50 in
Since the structure is linear and elastic, the required increase of length for each section will
be
3.0
0.50
(0.1) = 0.60 in
If we use the practical value of 0.625 in., the theoretical camber will be
(6.25)(0.50)
0.1
= 3.125 in
Victor E. Saouma; CVEN 3525; Univ. of Colorado Virtual Work 23/23
V. Saouma; CVEN‐3525 1
Solve for the vertical displacement at C of the following structure.
Step 1. Draw free body diagram and calculate all forces due to real load and virtual load .
A B C
80 160
D
2k
yD
xD
xA
yA
(CCW +ve) 0
(60) 2(240) 0
0
0
0
2
8
0 (1)
8
A
x
x
x x
y
y y
x
x
M
D
F
D A
F
A D
D k
A k
+
Σ =
− =
→Σ =
− + =
+ ↑ Σ =
− =
=
− +
=
A B C
2k
80 160
3
4
D
2 7
Aluminum rod, A =1 in ,E =10 psi
2 4 7
Aluminum Beam, A =10 in , I = 171 in ,E =10 psi
60
REACTIONS AND DISPLACEMENTS
V. Saouma; CVEN‐3525 2
D
yD
8k 3
4
T
3
(2)
5
4
1
5
0
8
y
x
T D
T
k
D k
T
=
=
=
=
.(2)
From .(
4
1)
6
A
y
y
From Eq
Eq
D k
k
=
=
Likewise, we can get the reaction due to unit virtual load at point C by similar manner.
A B C
80 160
2k6k
8k
8k
4k
A B C
80 160
1Pδ =3k
4k
4k
2k
Free Body Diagram due to real load Free Body Diagram due to virtual load
160”
V. Saouma; CVEN‐3525 3
Element x=0 P δP
DB D 10 5
Step 2. Determine forces in cable and beam for each elements due to real load and virtual 
load
(i) Cable. 
Tension is constant along the cable length. 
(ii)    Beam. 
A B C
80 160
2k6k
8k
8k
4k
A B C
80 160
1Pδ =3k
4k
4k
2k
Free Body Diagram due to real load Free Body Diagram due to virtual load
Element x=0 P δP M δM
AB A ‐8 ‐4 ‐4x ‐2x
BC B 0 0 2x‐320 x‐160
You may not get something as complex
for deflection; however make sure that
you understand this.
V. Saouma; CVEN‐3525 4
From (i) and (ii)
*
*
100 80
0 0
80 160
0 0
100 80
2 7 2 7
0 0
(5)(10) ( 4)( 8)
(1)
( 2 )( 4 ) ( 160)(2 320)
(5)(10) ( 4)( 8)
(1 )(10 ) (10 )(10 )
( 2
C
L L
w
u
C
c b
b b
P M
P P dx M dx
AE EI
dx dx
A E A E
x x x x
dx
EI EI
dx dx
in psi in psi
x
δ
δ
δ δ δΔ = +
− −
Δ = +
− − − −
+ +
− −
= +
−
+
∫ ∫
∫ ∫
∫ ∫
∫ ∫
80 160
7 4 7 4
0 0
)( 4 ) ( 160)(2 320)
(10 )(171 ) (10 )(171 )
0.0005 0.0000256 0.0007984 0.001597
0.0005256 0
0
.
.
00
00
239
)
54
29 (
C
Axial Contribution Flexure Contrib t
C
u ion
x x x
dx
psi in p i i
n
n
i
s
− − −
+
Δ = + + +
= +
Δ = ↓
∫ ∫
V. Saouma; CVEN‐3525 10
For the truss shown below:
a. Determine the horizontal deflection at C, Assume AE constant. 
A E
D
C
B
600 lb
1500 lb
8'
8'
8'
TRUSS DEFLECTIONS
V. Saouma; CVEN‐3525 11
(a). Determine the horizontal deflection at C. Assume AE constant.
A E
D
C
B
600 lb
1500 lb
8'
8'
8'
2
1
5
6
4
3
A E
D
C
B
1Pδ =
8'
8'
8'
2
1
5
6
4
3
Real load Virtual load
Member δP(lb) P(lb) L(in) AE δP.(PL/AE)
1 2 1,200 96 AE 230,400/AE
2 2 1,200 96 AE 230,400/AE
3 0 ‐1,500 48 AE 0
4 ‐2.24 ‐1,341.64 107.33 AE 322,560/AE
5 0 1,677.05 107.33 AE 0
6 ‐2.24 ‐3,018.69 107.33 AE 725,759.6/AE
1,509,119lb in
AE
−
∑ =
6
1.51 10
( )h
lb in
C
AE
× −
Δ = →
You will not get such a
complex truss, however
the procedure is very
important to understand
V. Saouma; CVEN‐3525 12
(b). Remove the loads and determine the horizontal displacement of C if members AB and  BC 
experience a temperature increase ΔТ=200°F, Take A = 2 in2, E = 29,000 ksi, and α=10‐6/°F
Member L(in) α(1/°F) ΔT(°F) ΔL=αΔTL(in) δP δP.ΔL
1 96 10‐6 200 0.0192 2 0.0384
2 96 10‐6 200 0.0192 2 0.0384
3 48 10‐6 0 0 0 0
4 107.33 10‐6 0 0 ‐2.24 0
5 107.33 10‐6 0 0 0 0
6 107.33 10‐6 0 0 ‐2.24 0
0.0768∑ =
0.0768 ( )h
C inΔ = →
A E
D
C
B
2
1 5
6
4
3200T FΔ =
You will not get such a
complex truss, however
the procedure is very
important to understand
V. Saouma; CVEN‐3525 13
c. Remove the loads and determine the horizontal displacement of C if member CD is 
fabricated 0.5 in too short.
A E
D
C
B
2
1 5
6
4
3
Fabricated 0.5 in 
too short.
Member L(in) ΔL(in) δP δP.ΔL
1 96 0 2 0
2 96 0 2 0
3 48 0 0 0
4 107.33 ‐0.5 ‐2.24 1.12
5 107.33 0 0 0
6 107.33 0 ‐2.24 0
1.12∑ =
1.12 ( )h
C inΔ = →
You will not get such a
complex truss, however
the procedure is very
important to understand

More Related Content

Similar to Review fe-exam-structures-saouma

Presentation_Final_Amrit - Ready to Present.pptx
Presentation_Final_Amrit - Ready to Present.pptxPresentation_Final_Amrit - Ready to Present.pptx
Presentation_Final_Amrit - Ready to Present.pptxNolarajPoudel
 
DEFORMACIONES ANGULARES - RESISTENCIA DE LOS MATERIALES
DEFORMACIONES ANGULARES - RESISTENCIA DE LOS MATERIALES DEFORMACIONES ANGULARES - RESISTENCIA DE LOS MATERIALES
DEFORMACIONES ANGULARES - RESISTENCIA DE LOS MATERIALES JHON ANYELO TIBURCIO BAUTISTA
 
Page 6 of 8Engineering Materials ScienceMetals LabLEEDS .docx
Page 6 of 8Engineering Materials ScienceMetals LabLEEDS .docxPage 6 of 8Engineering Materials ScienceMetals LabLEEDS .docx
Page 6 of 8Engineering Materials ScienceMetals LabLEEDS .docxbunyansaturnina
 
1 s2.0-s0997753817308902-main
1 s2.0-s0997753817308902-main1 s2.0-s0997753817308902-main
1 s2.0-s0997753817308902-mainFares Tounsi
 
manual-pe-2017_compress.pdf
manual-pe-2017_compress.pdfmanual-pe-2017_compress.pdf
manual-pe-2017_compress.pdfAbdo Brahmi
 
Modeling of the damped oscillations of the viscous
Modeling of the damped oscillations of the viscousModeling of the damped oscillations of the viscous
Modeling of the damped oscillations of the viscouseSAT Publishing House
 
"Deep Learning" Chap.6 Convolutional Neural Net
"Deep Learning" Chap.6 Convolutional Neural Net"Deep Learning" Chap.6 Convolutional Neural Net
"Deep Learning" Chap.6 Convolutional Neural NetKen'ichi Matsui
 
Library Characterization Flow
Library Characterization FlowLibrary Characterization Flow
Library Characterization FlowSatish Grandhi
 
Mit2 092 f09_lec01
Mit2 092 f09_lec01Mit2 092 f09_lec01
Mit2 092 f09_lec01Rahman Hakim
 
Sag tension calculations-a_tutorial_deve
Sag tension calculations-a_tutorial_deveSag tension calculations-a_tutorial_deve
Sag tension calculations-a_tutorial_deveRaghavTripathi11
 

Similar to Review fe-exam-structures-saouma (20)

ED7104 VAC_notes
ED7104 VAC_notesED7104 VAC_notes
ED7104 VAC_notes
 
Presentation_Final_Amrit - Ready to Present.pptx
Presentation_Final_Amrit - Ready to Present.pptxPresentation_Final_Amrit - Ready to Present.pptx
Presentation_Final_Amrit - Ready to Present.pptx
 
deformaciones angulares - vigas conjugadas
deformaciones angulares -  vigas conjugadasdeformaciones angulares -  vigas conjugadas
deformaciones angulares - vigas conjugadas
 
DEFORMACIONES ANGULARES - RESISTENCIA DE LOS MATERIALES
DEFORMACIONES ANGULARES - RESISTENCIA DE LOS MATERIALES DEFORMACIONES ANGULARES - RESISTENCIA DE LOS MATERIALES
DEFORMACIONES ANGULARES - RESISTENCIA DE LOS MATERIALES
 
U01232170177
U01232170177U01232170177
U01232170177
 
Page 6 of 8Engineering Materials ScienceMetals LabLEEDS .docx
Page 6 of 8Engineering Materials ScienceMetals LabLEEDS .docxPage 6 of 8Engineering Materials ScienceMetals LabLEEDS .docx
Page 6 of 8Engineering Materials ScienceMetals LabLEEDS .docx
 
1 s2.0-s0997753817308902-main
1 s2.0-s0997753817308902-main1 s2.0-s0997753817308902-main
1 s2.0-s0997753817308902-main
 
manual-pe-2017_compress.pdf
manual-pe-2017_compress.pdfmanual-pe-2017_compress.pdf
manual-pe-2017_compress.pdf
 
Modeling of the damped oscillations of the viscous
Modeling of the damped oscillations of the viscousModeling of the damped oscillations of the viscous
Modeling of the damped oscillations of the viscous
 
"Deep Learning" Chap.6 Convolutional Neural Net
"Deep Learning" Chap.6 Convolutional Neural Net"Deep Learning" Chap.6 Convolutional Neural Net
"Deep Learning" Chap.6 Convolutional Neural Net
 
Library Characterization Flow
Library Characterization FlowLibrary Characterization Flow
Library Characterization Flow
 
CE541-F14.Bhobe.Jain
CE541-F14.Bhobe.JainCE541-F14.Bhobe.Jain
CE541-F14.Bhobe.Jain
 
Mit2 092 f09_lec01
Mit2 092 f09_lec01Mit2 092 f09_lec01
Mit2 092 f09_lec01
 
Cme434 project
Cme434 projectCme434 project
Cme434 project
 
ansys tutorial
ansys tutorialansys tutorial
ansys tutorial
 
Maxwell3 d
Maxwell3 dMaxwell3 d
Maxwell3 d
 
Maxwell3 d
Maxwell3 dMaxwell3 d
Maxwell3 d
 
Maxwell 3D
Maxwell 3DMaxwell 3D
Maxwell 3D
 
Sag tension calculations-a_tutorial_deve
Sag tension calculations-a_tutorial_deveSag tension calculations-a_tutorial_deve
Sag tension calculations-a_tutorial_deve
 
Cfx12 09 turbulence
Cfx12 09 turbulenceCfx12 09 turbulence
Cfx12 09 turbulence
 

Recently uploaded

MANUFACTURING PROCESS-II UNIT-5 NC MACHINE TOOLS
MANUFACTURING PROCESS-II UNIT-5 NC MACHINE TOOLSMANUFACTURING PROCESS-II UNIT-5 NC MACHINE TOOLS
MANUFACTURING PROCESS-II UNIT-5 NC MACHINE TOOLSSIVASHANKAR N
 
(ANJALI) Dange Chowk Call Girls Just Call 7001035870 [ Cash on Delivery ] Pun...
(ANJALI) Dange Chowk Call Girls Just Call 7001035870 [ Cash on Delivery ] Pun...(ANJALI) Dange Chowk Call Girls Just Call 7001035870 [ Cash on Delivery ] Pun...
(ANJALI) Dange Chowk Call Girls Just Call 7001035870 [ Cash on Delivery ] Pun...ranjana rawat
 
Booking open Available Pune Call Girls Koregaon Park 6297143586 Call Hot Ind...
Booking open Available Pune Call Girls Koregaon Park  6297143586 Call Hot Ind...Booking open Available Pune Call Girls Koregaon Park  6297143586 Call Hot Ind...
Booking open Available Pune Call Girls Koregaon Park 6297143586 Call Hot Ind...Call Girls in Nagpur High Profile
 
Call Girls Pimpri Chinchwad Call Me 7737669865 Budget Friendly No Advance Boo...
Call Girls Pimpri Chinchwad Call Me 7737669865 Budget Friendly No Advance Boo...Call Girls Pimpri Chinchwad Call Me 7737669865 Budget Friendly No Advance Boo...
Call Girls Pimpri Chinchwad Call Me 7737669865 Budget Friendly No Advance Boo...roncy bisnoi
 
MANUFACTURING PROCESS-II UNIT-1 THEORY OF METAL CUTTING
MANUFACTURING PROCESS-II UNIT-1 THEORY OF METAL CUTTINGMANUFACTURING PROCESS-II UNIT-1 THEORY OF METAL CUTTING
MANUFACTURING PROCESS-II UNIT-1 THEORY OF METAL CUTTINGSIVASHANKAR N
 
University management System project report..pdf
University management System project report..pdfUniversity management System project report..pdf
University management System project report..pdfKamal Acharya
 
VIP Call Girls Ankleshwar 7001035870 Whatsapp Number, 24/07 Booking
VIP Call Girls Ankleshwar 7001035870 Whatsapp Number, 24/07 BookingVIP Call Girls Ankleshwar 7001035870 Whatsapp Number, 24/07 Booking
VIP Call Girls Ankleshwar 7001035870 Whatsapp Number, 24/07 Bookingdharasingh5698
 
BSides Seattle 2024 - Stopping Ethan Hunt From Taking Your Data.pptx
BSides Seattle 2024 - Stopping Ethan Hunt From Taking Your Data.pptxBSides Seattle 2024 - Stopping Ethan Hunt From Taking Your Data.pptx
BSides Seattle 2024 - Stopping Ethan Hunt From Taking Your Data.pptxfenichawla
 
Call for Papers - Educational Administration: Theory and Practice, E-ISSN: 21...
Call for Papers - Educational Administration: Theory and Practice, E-ISSN: 21...Call for Papers - Educational Administration: Theory and Practice, E-ISSN: 21...
Call for Papers - Educational Administration: Theory and Practice, E-ISSN: 21...Christo Ananth
 
Introduction to Multiple Access Protocol.pptx
Introduction to Multiple Access Protocol.pptxIntroduction to Multiple Access Protocol.pptx
Introduction to Multiple Access Protocol.pptxupamatechverse
 
UNIT-III FMM. DIMENSIONAL ANALYSIS
UNIT-III FMM.        DIMENSIONAL ANALYSISUNIT-III FMM.        DIMENSIONAL ANALYSIS
UNIT-III FMM. DIMENSIONAL ANALYSISrknatarajan
 
MANUFACTURING PROCESS-II UNIT-2 LATHE MACHINE
MANUFACTURING PROCESS-II UNIT-2 LATHE MACHINEMANUFACTURING PROCESS-II UNIT-2 LATHE MACHINE
MANUFACTURING PROCESS-II UNIT-2 LATHE MACHINESIVASHANKAR N
 
UNIT-II FMM-Flow Through Circular Conduits
UNIT-II FMM-Flow Through Circular ConduitsUNIT-II FMM-Flow Through Circular Conduits
UNIT-II FMM-Flow Through Circular Conduitsrknatarajan
 
ONLINE FOOD ORDER SYSTEM PROJECT REPORT.pdf
ONLINE FOOD ORDER SYSTEM PROJECT REPORT.pdfONLINE FOOD ORDER SYSTEM PROJECT REPORT.pdf
ONLINE FOOD ORDER SYSTEM PROJECT REPORT.pdfKamal Acharya
 
Call Girls in Nagpur Suman Call 7001035870 Meet With Nagpur Escorts
Call Girls in Nagpur Suman Call 7001035870 Meet With Nagpur EscortsCall Girls in Nagpur Suman Call 7001035870 Meet With Nagpur Escorts
Call Girls in Nagpur Suman Call 7001035870 Meet With Nagpur EscortsCall Girls in Nagpur High Profile
 
Structural Analysis and Design of Foundations: A Comprehensive Handbook for S...
Structural Analysis and Design of Foundations: A Comprehensive Handbook for S...Structural Analysis and Design of Foundations: A Comprehensive Handbook for S...
Structural Analysis and Design of Foundations: A Comprehensive Handbook for S...Dr.Costas Sachpazis
 
High Profile Call Girls Nagpur Isha Call 7001035870 Meet With Nagpur Escorts
High Profile Call Girls Nagpur Isha Call 7001035870 Meet With Nagpur EscortsHigh Profile Call Girls Nagpur Isha Call 7001035870 Meet With Nagpur Escorts
High Profile Call Girls Nagpur Isha Call 7001035870 Meet With Nagpur Escortsranjana rawat
 
Introduction to IEEE STANDARDS and its different types.pptx
Introduction to IEEE STANDARDS and its different types.pptxIntroduction to IEEE STANDARDS and its different types.pptx
Introduction to IEEE STANDARDS and its different types.pptxupamatechverse
 

Recently uploaded (20)

MANUFACTURING PROCESS-II UNIT-5 NC MACHINE TOOLS
MANUFACTURING PROCESS-II UNIT-5 NC MACHINE TOOLSMANUFACTURING PROCESS-II UNIT-5 NC MACHINE TOOLS
MANUFACTURING PROCESS-II UNIT-5 NC MACHINE TOOLS
 
(ANJALI) Dange Chowk Call Girls Just Call 7001035870 [ Cash on Delivery ] Pun...
(ANJALI) Dange Chowk Call Girls Just Call 7001035870 [ Cash on Delivery ] Pun...(ANJALI) Dange Chowk Call Girls Just Call 7001035870 [ Cash on Delivery ] Pun...
(ANJALI) Dange Chowk Call Girls Just Call 7001035870 [ Cash on Delivery ] Pun...
 
Booking open Available Pune Call Girls Koregaon Park 6297143586 Call Hot Ind...
Booking open Available Pune Call Girls Koregaon Park  6297143586 Call Hot Ind...Booking open Available Pune Call Girls Koregaon Park  6297143586 Call Hot Ind...
Booking open Available Pune Call Girls Koregaon Park 6297143586 Call Hot Ind...
 
Call Girls Pimpri Chinchwad Call Me 7737669865 Budget Friendly No Advance Boo...
Call Girls Pimpri Chinchwad Call Me 7737669865 Budget Friendly No Advance Boo...Call Girls Pimpri Chinchwad Call Me 7737669865 Budget Friendly No Advance Boo...
Call Girls Pimpri Chinchwad Call Me 7737669865 Budget Friendly No Advance Boo...
 
Roadmap to Membership of RICS - Pathways and Routes
Roadmap to Membership of RICS - Pathways and RoutesRoadmap to Membership of RICS - Pathways and Routes
Roadmap to Membership of RICS - Pathways and Routes
 
MANUFACTURING PROCESS-II UNIT-1 THEORY OF METAL CUTTING
MANUFACTURING PROCESS-II UNIT-1 THEORY OF METAL CUTTINGMANUFACTURING PROCESS-II UNIT-1 THEORY OF METAL CUTTING
MANUFACTURING PROCESS-II UNIT-1 THEORY OF METAL CUTTING
 
University management System project report..pdf
University management System project report..pdfUniversity management System project report..pdf
University management System project report..pdf
 
VIP Call Girls Ankleshwar 7001035870 Whatsapp Number, 24/07 Booking
VIP Call Girls Ankleshwar 7001035870 Whatsapp Number, 24/07 BookingVIP Call Girls Ankleshwar 7001035870 Whatsapp Number, 24/07 Booking
VIP Call Girls Ankleshwar 7001035870 Whatsapp Number, 24/07 Booking
 
BSides Seattle 2024 - Stopping Ethan Hunt From Taking Your Data.pptx
BSides Seattle 2024 - Stopping Ethan Hunt From Taking Your Data.pptxBSides Seattle 2024 - Stopping Ethan Hunt From Taking Your Data.pptx
BSides Seattle 2024 - Stopping Ethan Hunt From Taking Your Data.pptx
 
Call for Papers - Educational Administration: Theory and Practice, E-ISSN: 21...
Call for Papers - Educational Administration: Theory and Practice, E-ISSN: 21...Call for Papers - Educational Administration: Theory and Practice, E-ISSN: 21...
Call for Papers - Educational Administration: Theory and Practice, E-ISSN: 21...
 
Introduction to Multiple Access Protocol.pptx
Introduction to Multiple Access Protocol.pptxIntroduction to Multiple Access Protocol.pptx
Introduction to Multiple Access Protocol.pptx
 
UNIT-III FMM. DIMENSIONAL ANALYSIS
UNIT-III FMM.        DIMENSIONAL ANALYSISUNIT-III FMM.        DIMENSIONAL ANALYSIS
UNIT-III FMM. DIMENSIONAL ANALYSIS
 
MANUFACTURING PROCESS-II UNIT-2 LATHE MACHINE
MANUFACTURING PROCESS-II UNIT-2 LATHE MACHINEMANUFACTURING PROCESS-II UNIT-2 LATHE MACHINE
MANUFACTURING PROCESS-II UNIT-2 LATHE MACHINE
 
UNIT-II FMM-Flow Through Circular Conduits
UNIT-II FMM-Flow Through Circular ConduitsUNIT-II FMM-Flow Through Circular Conduits
UNIT-II FMM-Flow Through Circular Conduits
 
ONLINE FOOD ORDER SYSTEM PROJECT REPORT.pdf
ONLINE FOOD ORDER SYSTEM PROJECT REPORT.pdfONLINE FOOD ORDER SYSTEM PROJECT REPORT.pdf
ONLINE FOOD ORDER SYSTEM PROJECT REPORT.pdf
 
Call Girls in Nagpur Suman Call 7001035870 Meet With Nagpur Escorts
Call Girls in Nagpur Suman Call 7001035870 Meet With Nagpur EscortsCall Girls in Nagpur Suman Call 7001035870 Meet With Nagpur Escorts
Call Girls in Nagpur Suman Call 7001035870 Meet With Nagpur Escorts
 
Structural Analysis and Design of Foundations: A Comprehensive Handbook for S...
Structural Analysis and Design of Foundations: A Comprehensive Handbook for S...Structural Analysis and Design of Foundations: A Comprehensive Handbook for S...
Structural Analysis and Design of Foundations: A Comprehensive Handbook for S...
 
High Profile Call Girls Nagpur Isha Call 7001035870 Meet With Nagpur Escorts
High Profile Call Girls Nagpur Isha Call 7001035870 Meet With Nagpur EscortsHigh Profile Call Girls Nagpur Isha Call 7001035870 Meet With Nagpur Escorts
High Profile Call Girls Nagpur Isha Call 7001035870 Meet With Nagpur Escorts
 
Water Industry Process Automation & Control Monthly - April 2024
Water Industry Process Automation & Control Monthly - April 2024Water Industry Process Automation & Control Monthly - April 2024
Water Industry Process Automation & Control Monthly - April 2024
 
Introduction to IEEE STANDARDS and its different types.pptx
Introduction to IEEE STANDARDS and its different types.pptxIntroduction to IEEE STANDARDS and its different types.pptx
Introduction to IEEE STANDARDS and its different types.pptx
 

Review fe-exam-structures-saouma

  • 1. FE Exam Review for Structural Analysis Prof. V. Saouma Oct. 2013  Structural Analysis is part of the afternoon exam.  In the afternoon, you are to answer 60 questions, and Structural Analysis is about 10% of the test content (or about 6 questions).  Each question is worth 2 points.  You are expected to know: 1. Structural analysis of statically determinate beams, trusses and frames. 2. Deflection analysis of statically determinate beams, trusses and frames. 3. Stability analysis of beams, trusses and frames. 4. Column analysis (e.g. buckling, boundary conditions). 5. Loads and load paths (e.g. dead, live, moving). 6. Elementary statically indeterminate structures.  The only page in the “Supplied-Reference Handbook” related to Structural Analysis (shown in the next page).  Make sure that you know how to make best use of it, as it contains: 1. Reminder of what do we mean by “Moving Loads”. 2. Beam-Stiffness and moment carryover: to use for the analysis of statically indeterminate beams (unlikely that you get a SI frame). 3. Equations for the calculations of the deflections of trusses and beams using the virtual work method. Careful it is the virtual force/moment time the actual displacement (FL/AE for trusses, and M/EI for beams). 4. Member fixed end actions for uniform and concentrated load.  I strongly recommend that you also memorize: for the maximum deflection of a uniformly loaded, simply supported beam.  Careful with the SI units, GPa is 109 Pa or 109 N/m2 Many problems use the SI system.  In most cases, you will be dealing with round numbers, which greatly simplify your calculations.  Do not be tricked in believing that all triangles are 3-4-5.
  • 2.
  • 3. 26 (ivilDisdpline回 SpecificReviewfortheFE/EITEx目m FromthePVC,thelowpointislocatedat mA 吐 A せ十 m 一 m 的 町 一 つG A 斗 A F L一 A 吐 0 0一 A せ 一一一一 mA せ 4 パヨ+ L 一2 Determinethetangentoffsetぅ仏 atthelowpoint. y=iG2- G1)X 2 2L - (0.016一 (-0.02))(444m)2 (2)(800m) ニ 4.44m eleVlowpoint=742.12m十 4.44m =746.56m (747m) TheanswerisB. STRUCTURALANA町SIS Problem38 They-coordinate(mea師su町1 tむroぱidfortheT-shapedbeamis 「一「ーr---I J08m比 X y 0.23m (A) 0.08m (B) 0.16m (C)0却 m (D) 0.38m PPI・ 附W押i2pllss.com Solution Divide七hebeamin七otwoshapesasshown Al=(0.08m)(0.15m) =0.012m2 A2=(0.05m)(0.15m) ェ 0.0075m2 Thedis七ancefrom七hetopofsection2tothecentroidis Uc=Z生Yc,i 2乞:Aん4 (( 川 十((0.0075m 2 )(平十0叫)) ニ 0.08m TheanswerisA. Problem39 TheforceinmemberHEfromthe七russshownismos七 nearly G ~1 12.0m (A) 1110Ncompression (B) 1110Ntension (C) 2490Ncompression (D) 2490N七ension Solved Problems
  • 4. Solution 8ummomentsaboutAtofindtheverticalreactionatE. :Z=MA= 0 RE,匂 dEA RH:"=i2224N)(伊6m) U円'げν 1ロ2m =1112N Thefree-bodydiagramabou七pointEis 、....._.EH DE !RE,ν 8ummingforcesinthehorizontaldirectiongivesthe horizontalreactionatEas0N. Thesumofforcesin位1everticaldirectionis FEH,υ-RE,り=0lbf =1112Ncompression ThehorizontalcomponentoftheforceinmemberEHis FEH,h=(2)(1112N) =2224N Theresultan七forceinmemberEHis FEH=V(Fi凹,出(Fi叫2 =ゾ(1112N)2十 ( 山 N)2 ニ 2487Ncompre回 on (2490N∞mpr出品on) TheanswerisC. Problems40-42arebasedonthefollowinginformation andillustra七ion. Aplanetrussisloadedasshown. Pr目di(eProblems 27 Problem40 Themagnitudeoftheverticalreactionforceatsup- portAismostnearly (A) 3.3kN (B) 6.7kN (0) 10kN (D)16kN Solution 8incesupportDisarollersupportう thehorizontalre- ac七ionforceう RAx,is0kN.Tofindtheverticalreaction atsupportA,RAy,afree-bodydiagramisdrawnofthe entire七russandmomentsaresummedaboutsuppor七D. RAy(15m)-(20kN)(5m)=0kN RA"=i20kN)(5m) 吋 15m =6.67kN (6.7kN) TheanswerisB. Problem41 ThemagnitudeofthecompressiveforceinmemberAB ismostnearly (A)4.2kN (B) 6.7凶 (0) 8.5kN (D) 11kN Solution From801.40,thereactionatsupportA is6.67kN. Nextぅafree同 bodydiagr乱misdrawnforsupportAwith memberforc田 and七heirforcecomponents.Using七he Pythagoreantheorem,therelativemagnitudesofeach forceandeachforceうshorizontalandverticalcompo- m臨 canbefound. (Inthiscase,6.403,5,and4are therelativemagni七udesofthememberABforceand theforcesうshorizontalandverむicalcomponen七s,respec- tively.) FAEx PPI.附 w.ppi2p日sS.com
  • 5. 28 CivilDiscipline-Specifi(ReviewforfheFE/EITEx目m Forequilibrium,allforcesonafreebodymus七sumto okN.Summationofverticalforcesgiv<回 FABy+RAy=0kN Thiscanberearrangedtogive FABy= -RAy=-6.67kN Recallthatjointsin七russesarefrictionless,sonobend- ingmomentsexist. Theforceanditscomponentsareproportionaltothe geometriclengもhsofthetrianglesides. 一 (6.403m¥一 .l'AB= I一一了一一一一 I.l'AB'U ¥ 4m I ν (6.403m¥ =(τ'~--- )(-6.67kN) = -10.68kN (-11kN) Theanswerisnegative.Thismeansthatthecalculated forceisintheopposi七edirec七ionto七heassumedforcedi- rectiononthefree-bodydiagram.8incethememberAB forcew回 assumedtoapplytensiononthefree伊 bodydi- agram,thenegativeanswermeansthatmemberABis mcompreSSlOn. TheanswerisD. Problem42 ThemagnitudeoftheforceinmemberEFismos七ne乱rly (A)4.2kN (B) 5.3kN (C) 6.7kN (D)8.3悶 Solution From801.41,FAByis-6.67 日~. Thehorizontalcom- ponentofthememberABforceis M H L 且 可 tn h v U F O E U ρ h u 凡 ト N ¥ 1 1 1 1 / ¥、 1 1 1 I / L ι m 一 m m 一 m お v h u 一A 住 民 U 一an笹 口 δ / e l l - ¥ / 1 1 1 1 ¥ 一 一 一 一 一 一 一 ZBAF ForequilibriumatsupportA,thesumofthehorizontal forcesmustbeequalto0kN.TheforceinmemberAEis FAE=FAEx = -FABx =-(-8.33kN) =8.33kN PPI.附 W押i2p日ss.com Theposi七ivesigninthecalculatedmemberAEforce meansthattheassumeddirectionoftheforceon七he free-bodydiagramう whichindicatestension,isinthe samedirectionasthecalculatedforce. Afree-bodydiagramofjointEwillshowthatthever- ticalmemberBEisunabletosustainanyhorizontal force. Therefore,theforceinmember~F isthesame astheforceinmemberAE. FEF=FAE =8.33kN (8.3kN) TheanswerisD. Problem43 Ifthetrussmembersaremadeofsteelandthecross- sectionalareaofeachmemberis1000mm2 ,themagni- tudeoftheverticaldefl.ection抗 jointEismostnearly (A) 0.70mm (B) 1.1mm (C) 1.6m m (D)2.8mm Solution Usetheprincipleofvi巾 alwork(unitloadmethod). Theactualforcesineachmembercanbedetermined byapplyingtheequationsofequilibriumtoeachtruss jointandareshown回 follows. (80meround-offerror existsinthesecalculatednumbers.) τ'russmemberlengthsinmetersareasfollows. B 5 C AT< 41 々ず'" 1 4 仏 / I 、 士 一 '::"'D 5 E 5 F Theactualforcesinkilonewtonsareshown. 20 D 6.67 13.33
  • 6. PradiceProblems 29 Problems44and45arebasedonthefollowingillustra司 tion. Applicationofaverticalunitloadatjoi凶 E results inthevirtualmemberforcesinkilonewtonsasfollows. (Someroundoffeロorexis匂 inthesecalculatednum- bers.) 町5m ↑ 5m 手5m JD Problem44 Themagnitudeofthever七icalreac七ionforceatsup- port.Aismostnearly 0.33 Itisrecommendedthatatablebeusedtokeepallvari- ablesorganized.Usenegativevaluestorepresentcom- presslOn. 0.67 1.3kN 5.0kN 13kN 20kN (A) (B) (C) (D) FQFpL (kN2 岨) L (m) FQ (kN) Fp (kN) Solution Sincesuppor七Cisarollersuppor七ぅ thereisnohorizon- talreactionforceatthatpoint. Theverticalreaction forceatsupportAう RA約 canbefoundbyconverting theuniformlydistributedload,ω,intoaresultantpoint load. mv h u ¥、、 E B﹄ t /ノ 凶一 m v h u / I I I -、¥ 二 到 L i ω 引制 一 一 一 一 W member 73.1 35.0 72.4 34.6 34.6 35.0 0.0 8.8 -36.2 I:=239.7 6.4 5.0 6.4 5.0 5.0 5.0 4.0 4.0 6.4 -1.07 -0.42 0.53 +0.83 +0.83 十0.42 +1.0。+0.33 -0.53 10.67 -16.67 -21.34 十8.33 十8.33 +16.67 十0.00 -6.67 +10.67 B C D E F D E F F A B C A E F B C B Thisresul七antloadislocateda七thecentroidoftheuni- formlydistributedload. ThemodulusofelasticityofsteelisE=2.1X 1011 Pa. Sincetheareaandmodulusofelasticityarethesame foralltrussmembers,七heirproduct,AEう iscommonto allmembersandcanbetakenoutsideofthesummation forsimpli五cation. Thereforβ,theverticaldeflectionat pointEcanbefoundby r山 一 E H 一A ? Q L E A い P ε v b z M ¥ l / R l 一E Z 仁 い EA 5m ThesupportAverticalreactioncanthenbefoundby summingmomentsaboutsupportC.Thisresultsina verticalreactionatsupportAof 5m(1000mm 2 )(吋:m) × 川1Pa)(品) RAy(10m)-(15kN)(5m) +(25kN)(2.5m)=0kN-m x(1000誓) (1.1mr吋 ISincetheunitloadinthevirtualforcesystemwasdown- .wardandtheanswerispositiveinsign,theactualde- 丑ectionisalsodownward. RA"ニ (15kN)(5m)-(25kN)(2.5m) 吋 10m (1.3kN) TheanswerisB. PPI.附 W押i2p日sS.com = 1.25kN ェ1.14rnrn
  • 7. 30 CivilDiscipline-SpecificReviewfortheFE/EITExam Sincetheanswerispositiveinsign,thedirec七ionofthe calculatedreactionisthesameasthatoftheassumed reactionithatis,thedirectionofthereactionisupw乱rd. TheanswerisA. Problem45 Themagnitudeofthemaximumverticalshearinthe beamismostnearly (A) 1.3kN (B) 14kN (C) 25kN (D) 39kN Solution Oneway七odeterminetheanswertothisproblemisto constructasheardiagram.Thechangeinshearisthe areaundertheappliedloading. Althoughamomen土 diagramisnotrequiredforthisproblemう itfollowsthat thechangeinmomen七istheareaunderthesheardia- gram,soamomentdiagramisusuallyincluded. / llF15kN LkN/mI 1c~ ~ ~ ~ ~ ~ V(kN) M(kN'm)0 -62.5 Ascanbeseen,themaximumvalueofverticalshearis 25kNatsupportC. TheanswerisC. PPI・ 附W押i2p日sS.com STRUCTURALDESIGN Problems46and47arebasedonthefollowinginforma- tionandillustration. Thecrosssectionofareinforcedconcretebeamwith tensionreinforcementisshown.Assumethatthebeam isunderreinforced. 五=3000lbfjin 2 ん=40,000lbfjin 2 As=3in2 [threeno.9bars] 20in 23in Problem46 InaccordancewithAmericanConcreteInstitute(ACI) 凶rengthdesign,theallowablemomentcapaciもyof七he beamismostnearly (A) 160ft-kips (B) 180ιkips (C) 200ft-kips (D)210ft-kips Solution 一n 2 i 一如 - { 3 一 m 一 q G 一 1 i ニ お ーー ん一 M ω 一一一一 ρ' Inanactualdesignandanalysissituation, 乱 check shouldalways'bemadetoseeth乱,ttheactualreinforc- ingsteelratiofallsbetweentheallowablemaximumand allowableminimumsteelratios,eventhoughthischeck isnotrequiredtosolvethisspecificproblem.
  • 8. 46 CivilDiscipline-SpecificReviewfortheFE/EITExam Theminimumrequiredlengthofspiraltransitionbe- tweenthecurveandroadismostnearly (A) 28ft (B) 36此 (0)44氏 (D)72ft 36.Thedesignrequirementsforasectionofhighway witha1.5%gradeareasfollows. designspeed=80kmjh coefficientoffriction=0.35 driverreactiontime= 2.0s drivereyeheight=1.2m object(tobe師 oided)height= 0.2m Thedownhi1ldesignbrakingdistanceforthishighway ismostnearly (A)45m (B) 75m (0) 100m (D) 120m 37.Acrestonasectionofhighwayconsistsofavertical curvewitha1500mradiusandaposi七ive1%gradefol- lowedbyanegative3%grade.Thedesignrequirements areasfollows. designspeed=80kmjh drivereyeheight= 1.2m object(tobeavoided)height=0.2m 日七oppingsightdistanceデ300m Theminimumrequiredlengthofverticalcurveneeded tosa七isfythedesignstoppingsightdis七anceismost nearly (A) 680m (B) 700m (0) 760m (D) 840m 38.A superpavedesignmixtureforahighwaywith ESALs<107 h回 anominalmaximumaggregatesize of19mm. Themixturehasbeentestedandhasthe followingcharacteristics: airvoids=4.0% VMAニ 13.2% VFA= 70% dust-to-asphaltratio= 0.97 atN = 8gyrationsう Gm mニ 87.1% atN =174gyrationsぅ Gm m=97.5% PPI・ 附w.ppi2pass.com Dothesecharacteristicssatisfy七heircorrespondingsu- perpaverequirements? (A) Yes,allthep釘 ametersarewithin組 acc叩七回 ablerange. (B) No,theVMAisexcessive (0) No,thedust-to田 asphaltratioistoohigh (D) No,Gm matNm日 istoohigh. 39.A roadleadingtoas七onequarryistraveledby 40trucks,wi七heachtruckmakinganaverageof10trips perday. Whenfullyloaded,eachtruckconsis七sofa frontsingleaxletransmittingaforceof10,000lbfand tworeartandemaxlesう eachaxletransmittingaforce of20,000lbf.Theloadequivalencyfactorforthefront singleaxleis0.0877. Theloadequivalencyf配 torfor eachreartandemaxleis0.1206. The18,000lbfequivalentsingleaxleload(ESAL)for thetrucktra伍conthisroadfor5yrismostnearly (A) 0.33ESAL (B) 130ESAL (0) 48,000ESAL (D) 240,000ESAL Problems40and41町 ebasedonthefollowingi1lus七ra- tion. ↓ド15kN i「↓51NK FJT:;矛;;「 40.Themagnitudeof七hemaximumbendingmoment inthebeamismostnearly (A)6.3kN-m (B) 14凶 m (0) 25kN.m (D)63kN-m 41.Ifthebeamismadeentirelyofsteelandthewhole beamhasamomentofinertiaabouttheaxisofbending of2.0X 108 mm4, themagnitudeoftheverticaldefl_ec- tionatpointDismostnearly (A) 0.20mm (B) 2.3m m (0) 23m m (D)50mm Practice Exam I
  • 9. Prlldiceb:llm1 47 Problems42and43arebasedonthefollowinginforma- tionandillustra七ion. Atruckisfacinginitsintendeddirectionoftravelalong thebeamasshown. トx 下予 lfm 手ょ :iD 42.Foratrucktravelinginthedirectionshown,七he ma.ximumverもicalliveloadshearatsuppor七Cismost nearly (A)27kN (B) 76kN (C) 100kN (D) 110凶 43.Foratrucktravelinginthedirectionshownう the ma.ximumliveloadbendingmoment抗 supportCis mostne紅 ly (A)80kN-m (B) 90kN.m (C) 140kN.m (D)360kN.m Problems44and45arebasedDnthefollowinginforma- tionandillustration. Aplanetrussspanisshown.Theroadwaybehavesas simplysupportedbeamspansbetweenthesupporting lowerchordtrussjoints.Byconvention,positiveforces aretensileforcesう andnegativeforcesarecompressive forces. mA ﹃ I a T i l l - - 4 7「F 5m 5m 5m 44.Inthex-directionasshown,thema.ximuminflu- encelineordinatefortensileforceinmemberBFismost nearly (A) 0.18kN/kN (B) 0.36kN/kN (C) 0.53kN/kN (D) 0.71kN/kN 45.Inthex-directionasshown,thema.ximuminflu- encelineordinateforcompressiveforceinmemberBF ismostnearly (A) -0.71悶 /kN (B) -0.53kN/凶 (C) -0却 kN/kN (D) -0.18kN/kN Problems46and47arebasedonthefollowinginforma- tionandillustra七ion. Thecrosssectionsoftwoshort,concentricallyloaded reinforcedconcretecolumnsareshown. だ=4000Ibf/in2 fy= 60,000Ibf/in2 18in 18in longitundinal reinforcement roundspiralcolumn (crosssection) (Prob圃 46) squaretiedcolumn (crosssection) (Prob.47) 46.Fortheshortroundspiralcolumn,theapplieda.xial deadloadis150kips,andtheapplieda.xialliveloadis 350kips. Assumingthatthelongitudinalreinforcing barsareallthesamesize,theminimumrequiredsizeof eachlongitudinalreinforcingbaris (A) no.7 (B) no.8 (C) no.9 (D)no.10 47.Fortheshortsquaretiedcolumn,theapplieda.xial deadloadis150kips,andtheapplieda.xialliveload is250kips.Assumingtha七七helongitudinalreinforcing PPI.附 w.ppi2pass.com
  • 10. Themomentfunctionsinthedirectionsindicatedby the1oca1x-coordinateforeachbeamsegmentunderthe 乱ctua110adingareshownonthefollowingmomentdia同 41.Usetheprincip1eofvirtua1workto五nd七hevertica1 de丑ectionatpointD. 1S 40.Constructshearandmome凶 diagrams.Thechange inshearistheareaundertheapplied1oading,andthe changeinmome凶 istheareaunder七hesheardiagram. N│一:-1 札 rニ 州 江lOment (240,000ESAL) ムD =~(Jm(自白) -62.5 The 1argest magnitude of bending -62.5kN.m(63kN-m)atsupportC. =240う097ESAL For5yr,thetota1ESALis M 川I V A w h u ν I E q 4 J A - -q 川一 y ベー一 A h 一 一 円 円 ト L ﹄ 且 晶 mill-- A TheanswerisD. TheanswerisD. M{kN.m) 0 gram. V{kN) 39.Thetota1ESALpertruckforeachtripis ESAL七r叫 =(1sing1e砿 1e)(0.0877) +(2tandemax1es)(0.1206) =0.3289ESALjtruck-trip Thetota1dailyESALfor40trucks,eachmaking 10tripsaday,is Allparametersinthismixturearewithinsuperpave specifica七ions. Second,wherestoppingsigh七distanceisgreaterthan thecurve1eng七h: (200%)(VHi+♂'2/ L=2S A Since也eprevioustwoequationsshowthatthestop- pingsightdistanceis1essthanthecurve1ength,the minimumrequiredvertica1curve1engthis L=756.4m (760m) 38.Thismixturewou1dbedesignatedasa19m msu伺 perpavemixture.Thelimitsforsuchamixtureare airvoids=4.0% minimumVMλ=13% VFA=65-75% dusιto-aspha1tratio=0.6-1.2 atNinit=8gyratiorおう maximumGm m=89% atNmax=174gyrations,maximumGm m=98% First,wheres七oppingsigh七distance,8,is1essthanthe curve1engthう L: (200%)(Vl五五+VO万五:)2 =(2)(300m)一 1%-(-3%) L= A8 2 - (100%)(VWi+伊豆;y (1%一 (-3%))(300m)2 一 (100%)( 何 日 早 川 南 可f CivilDisdpline-SpecificReviewfortheFE/日TEx田町1 E叫 = 山cks)(10寄) X(0印刷)truckωtrip =131.56ESALjday 3'7.Therearetwoequa七ionstocheck. TheanswerisA. TheanswerisC. =756.4m PPI.州 w.ppi2pass.com = 481.0m 56 Solution Practice Exam I
  • 11. X X M(kN.m} 0 M =6.25- Themomentfunctionsin七hedirectionsindicatedbythe localx-coordinateforeachbeamsegmentunderthevir- tualunitloadingatpointDareshownonthefollowing momen七diagram. Atablesummarizesthemomentfunctionsasfollows. reglOn regron reglOn A-B B同 C C自 D function 0<x<5 0<x<5 0<x<5 M 1.25x 6.25-13.75x -2.5x2 m -0.5x -2.5-0.5x -x EI EI EI EI Pr目cticeExam1Solufions 57 ThemodulusofelasticityofsteelisE=2.1X 1011Pa. Fromtheprincipleofvirtualwork,theverticaldefl.ec- tionatpoin七Dis ムD =15 (- EI +J EI イー云 2 呼判:+一山1: I EI 十半1:十平1: 十年1: -26.042kN2.m3-78.125kN2.m3 十 390.625 日~2.m3 +286.458kN2.m3 十390.625kN2.m3 (2.1X 10 11 Pa)(品) / 紅l立1¥ XIllJlJU-一一ー l ¥ m / =22.9mm (23mm) Sincetheunitloadinthevirtualforcesystemwasdown- wardandtheanswerispositiveinsign,theactualde- fl.ectionisalsodownward. TheanswerisC. PPI.附 w.ppi2poss.com
  • 12. 58 CivilDiscipline-SpecificReviewfortheFEjEITExam 42.Constructanduseanin丑uencelineforverもicalshear atsupportC.Thein乱uencelineisconstructedbyplot- tingthechangeinresponseonafree四 bodydiagramof asectionofbeamatsuppo吋 Casaunitloadtravels acrossthestructure. 。 745m 5m influencelineforVc(kN/kN) loadposition formaximum verticalshear 1.5 t t71.17kN 17.79kN 1 _ 1 Theloadpositionshownresultsin出 emaximumre司 sponseinthebeamatsuppor七Cfor出especi五eddi- rectionoftravel. Themaximumverticalshearcanbe foundbysuperposition回 日 x =(1.5~~) (71.17凶) Thisproblemaskedforthemagnitudeofmaximumver- ticalshearatsupportCforatrucktnαvelinginthe directionshown.Ifthisproblemhadaskedforthemag- nitudeofmaximumverticalshearatsupportCforthe giventruckαxleconfigur,αtion,theaxleloadposi七ions wouldhavetobeswitchedaroundandbothaxleloads placedontheinfiuencelinewiththeheavieraxleload onthelargerinfiuencelineordinateandthelighteraxle loadonthesmallerin丑uencelineordinate.Thiswould resultinalargernumericalanswer. TheanswerisD. 43.Constructanduseaninfiuencelineforbendingmo- ment抗日uppo凶 C.Thein丑uencelineisconstructedby plottingthechangeinr田 ponseona丘ee四 bodydiagram ofasectionofbeamatsupportCasaunitloadtravels across七hestructure. PPI.附 w.ppi2p日sS.com 。 loadposition formaximum bendingmoment t t Theloadpositionshowngivesthemaximumresponse inthebeamatsuppor七Cforthespeci五eddirec七ionof travel. Themaximumbendingmomentcanbefound bysuperposition出 時 max= 1(-5守)川 = 355.8kN-m Thisproblemaskedforthemagnitudeofmaximum bendingmomentatsupportCforatrucktravelingin thedirectionshoωn.Ifthisproblemhadaskedforthe magnitudeofmaximumbendingmomentatsupportC forth巴 giventruckαxleconfigurαtionう theaxleloadpo- sitionswouldhavetobeswitchedaroundandbothaxle loadsplacedontheinfiuencelinewiththeheavieraxle loadonthelargerin宜uencelineordinateandthelighter axleloadonthesmallerin丑uencelineordinate.This wouldresultinalargernumericalanswer. TheanswerisD. 44.Constructaninfiuencelinefortheforceinmem- berBF.Thein丑uencelineisconstructedbyplotting thechangeinresponseinmemberBFasaunitload travelsacrossthestructure. _J.,"
  • 13. ゆ E ; f d務 lllJzrJ iiλ哩~I(附/kJ Whenmovingtheuni七loadacrossatrussstructure,this loadmustbedisむibutedtothetwojoints.Itisstated 七hattheroadwayactsassimplebeamspansbetween trussjoin七s.Therefore,whenaloadisplacedbetween trussjoin七s,thetwojointsadjoiningthebeamspanact assimplebeamsupportsandthemagni加 desofthe loadsappliedto出etwotrussjointsarethesameasfor thecalculatedreactionforcesof七hisbeam. ThemaximumordinatefortensileforceinmemberBF is0.53kN/kN TheanswerisC. 45.Fromtheinfl.uencelinein801.44,themaximum ordinateforcompressiveforceinmemberBFis-0.53 kN/kN TheanswerIsB. 46.Determinetheamountofreinforcings七eelrequired bytheminimumrequiredreinforcementratio,Pg,of 0.01.Theminimumareaofreinforcings七eelrequiredis =(0川(年) Determinetherequiredamountofreinforcingsteelbased onthefactoredaxialload,Pu. Puニ1.2Pdead+1.6P!ive ヱ(1.2)(150kips)+(1.6)(350kips) = 740kips Thenominalaxialcompressiveloadcapacityisgivenby 凡 =0.85P.。 = (0.85)(0.85f~Aconcrete +んAs) = (0部 )(0.85五(Ag-As)+んAs) Pr目di(eExam1Solutiolls S9 I七isrequiredthatゆPn2:Pu. Foraxialcompression withspiralreinforcement,ゆ=0.70.8ettingゆPn=P,叫 andsolvingfortheareaoflongitudinalreinforcingsteel gIVes 山 ps)(10002) 485)(4000主)(年) 。主一(0.85)(4000~~;) As= 6.69in2,requiredforthegivenappliedaxialcom- pressiveloads,isgreaterthanAs= 2.54in2basedon theminimumallowedreinforcementratio,Pg=0.01. TheminimumrequiredareaofreinforcementisAs二 6司 69in2. Thecolumnhassixlongitudinalreinforcingbars.The requiredareaofeachlongi七udinalreinforcingbaris A=~ 一日主主一 -nbars 6bars = 1.11in2 /bar A barareaof1.11in2issatisfiedbyano.10bar, whichhasanominalareaof1.27in2. Inanactualdesign/analysissituation,乱 checkshould alsobemadetoseethattheactuallongitudinalrein- forcementratiodoesnotexceedthemaximumallowable ratioof0.08. TheanswerisD. 47.Determinetheamountofreinforcingsteelrequired bytheminimumrequiredreinforcementratio,内, of 0.01.Theminimumareaofreinforcingsteelrequiredis As= pgAg = (0.01)(18in)2 =3.24in2 Determinetherequiredamoun七ofreinforcingsteelbased onthefactoredaxi,alloadう Pu. Pu=1.2Pde吋+1.6P!ive =(1.2)(150kips)+(1.6)(250kips) =580kips PPI.附 w.ppi2p日sS.com
  • 14. 70 CivilDiscipli鵬闘SpedficReviewfortheFE/EITEx脚 The七ra伍cflowrelationshipisgivenbyq=kv,inwhich qisthetra伍cvolumeinveh/hr.Themaximumtra伍c volumefor七hisroadismostnearly (A) 760veh/hr (B) 880veh/hr (C) 900veh/hr (D)960veh/hr 38.Thes七oppingsightdis七anceis430ftforadesign speedof50mphonasectionofhighway. Thegrades forthishighwaysectionare-1%followedby3%.The requiredlengthofverticalcurveneededtosatisfy七he AASHTOstoppingsightdistanceforthisdesignspeed ismostnearly (A) 270氏 (B) 380氏 (C)410ft (D)450ft 39.Aone-laneruralroadhasa100 curveextendingfor 230m alongitscenterline.Theroadis5mwidewith 3m wideshoulders.Thedesignspeedforthisroadis 75km/h. Thesuperelevationneededsothatsidefrictionisnot neededismostnearly (A) 0.00050 (B) 0.034 (C) 1.1 (D) 1.9 40.Thewornsurfacecourseofahigh-volumepave四 mentisbeingreplacedwithadesignrequiringatotal structuralnumberof6.6.Theengineerhasdecidedto replace6inofthesurfacewithrecycled-in-placeasphalt concretehavingasurfacecoursestrengthcoe血cientof 0.42,leavinginplace3inofsoundoriginalpavement havingastrengthcoefficientof0.3. Undertheorigi- nalpavementarea10inceme凶 -treatedbasehaving astrengthcoe血cientof0.20ぅ and阻 8insandygravel subbase.Whatistheminimumstrengthcoefficientfor thesubbase? (A) 0.05 (B) 0.10 (C) 0.15 (D)0.20 PPI. 州w.ppi2p日sS.com Problems41and42arebasedonthefollowinginforma- tionandillustration. Thebeamshownisloadedwithtwo1000Npointloads. Theseparationismaintained抗 2m,but出eloadsmay bemovedtoanylocationonthebe乱m. h 門 ⋮ 什 ﹁ 一 助 │ 蕩Z, 10m 10m 41.ThemaximumvalueforshearatsupportAismost nearly (A) 2000N (B) 2800N (C) 3000N (D)3800N 42.Themaximumvalueformomentatsuppor七A is mostnearly (A) 1.8kN.m (B) 8.0kN-m (C) 10kN.m (D)18kN-m 43.Atriangularpin-connectedtrusscarriesaloadof 4448Nasshown.Eachmemberhasthesamemodulus ofelasticityandcross-sectionalarea. :%:l M 3.0m P N 4448N 4.6m Practice Exam II
  • 15. ThetrussmemberpropertiesareE=200X 106 kPa andAニ 2580.6mm2.Theverticaldeflectionatpoint Pismostnearly (A)0.25mm (B) 0.48mm (C) 0.51mm (D)0.75mm Problems44-46arebasedonthefollowingillustration. l~ ~ ~ ~ ~ ~ ~ ~ ~ ~ ~ ~ ~ ~ L~ A 易 易 3 m 3 m 44.IfthereactionatsupportAis18.75N,thereaction ateachof出eoutersupportsismostnearly (A) 5.6N (B) 7.2N (C) 11N (D)14N 45.Themaximumvalueofverticalshearatanypoint alongthebeamismos七nearly (A)4.7N (B) 9.4N (C) 14N (D) 18N 46.Themaximumvalueofmomentatanypointalong thebeamismos七nearly (A) 1.2N.m (B) 3.1N.m (C) 4.6N.m (D)5.7N.m Pr目diceEx園田12 71 Problems47-49arebasedonthefollowinginformation andillustration. Thespanlengthandcrosssectionofareinforcedcon- cretebeamareshown. Thebeamisunderreinforced. Theconcreteandreinforcingsteelpropertiesぽ e五= 3000lbf/in 2 うん =40,000lbf/in 2 ,andAs=3in2 • 6ft 10ft 18i日目 beam cross section 47.Neglec七ingbeamself-weightandbasedonlyonthe allowablemomentcapacityofthebeamasdetermined usingAmericanConcreteInstitute(ACI)strengthde白 signspecificationsう themaximumallowableliveloadis mostnearly (A)23,000lbf (B) 29,000脳 (C) 35,000lbf (D)50,000lbf 48.Thebeamsupportsaconcentraもedliveloadof 50,000lbf.Neglec七 beamself-weight.Theminimum amoun七 ofshearreinforcementrequiredforacenter- to-centerstirrupspacingof12inunderACIstrength designspecificationsismostnearly (A)0.18in2 (B) 0.36in2 (C) 0.67in2 (D)0.78in2 49.Thebalancedreinforcingsteelratioforthisbeam inaccordancewithACIspecificationsismostnearly (A) 0.037 (B) 0.043 (C) 0.051 (D) 0.058 PPI.附 w.ppi2pass.com
  • 16. Thedensityis33.125veh/miformaximumtra缶cvol司 ume.Substitutionin七othetra白cflowrelationshipgives themaximumtra血cvolume. = (53::)(33.125:~) 一 (0.8品)(33.1叫 TheanswerisB. 38.Sincethereisanegativegradeprecedingapositive gradeう thisisasagverticalcurve.Usingthesagverti- calcurveequationsfromthecivilengineeringsectionof theNCEESHandbook,thealgebraicdifferencebetween gradesis A=1-1% 3%1 =4% Wherethestoppingsightdistance,8,isless七han七he verticalcurvelength,Lう L=-竺L- 400十 3.58 (4)(430ft)2 400十 (3.5)(430此) =388.2ft Wherethestoppingsightdistanceisgreaterthanthe curveleng七h, 400+3.58 L二 28一 一 A一 一 400+(3.5)(430氏) =(2)(430此) - 4 =383.8ft Fromthe七wovaluesforlengthofcurveう itcanbeseen thatthes七oppingsightdistanceisgreaterthan七he curvelength. Therefore,therequiredverticallength ofcurveis L=383.8抗 (380此) TheanswerisB. 39.Theradiusofcurvatureis R=~= 川寺) PradiceEx日m2501日tiol1S 81 Foraside-台ictionfactoroff=0,thesuperelevationis givenby ((75引(1吋(品)) (9.81~) (1318m) TheanswerisB. 40.Thestructuralnumberisthesumofproductsof thelayerdepths(thicknesses)andstrengthcoe血cients. SN=αrecycleDrecycle十 αoriginalsurfaceDoriginalsurface +αbaseDbase十 αsubbaseDsubbase 6.6=(0.42)(6in)+(0.3)(3in)十 (0.2)(10in) 十αsubbase(8in) αsubbase=0.15 TheanswerisC. 41.Drawanin丑uencelineforshearatsuppor七Awith loadspositionedasshownformaximumshearvalue. Forexampleう forshearatsupportAう usingbasicbeam statics, 20m 2.0 influencelineforshearatsupportA ト --1 一一一:51~20loadsplacedformaximumshearvalueatsupportA ThemaximumvalueforshearatsupportAis VA=(1000N)(1.8)+(1000N)(2.0) =3800N TheanswerisD. PPI.剛 w.ppi2p日sS.com Solution Practice Exam II
  • 17. 82 CivilDisdplille-SpedficReviewfortheFEjEITEx日m 42.AninfiuencelineforbendingmomentatsupportA canbedrawnwithloadspositionedasshownformax- imummome凶 value. Asin五nding七heinfiuenceline forshearう aunitloadismovedacrossthebeamandthe variationinbendingmomentatpointAisgraphed. 姦 10m す m A 10m influencelineformomentatsupportA loadsplacedformaximumvalueatsupportA ThemaximumvalueformomentatsupportA (dis- regardthesignぅ sincethemaximumvalueiswanted) lS MA= (1000N)(8.0m)+(1000N)(10.0m) 二 18000N.m (18kN.m) TheanswerisD. 43.Theprincipleofvirtualworkcanbeusedtofind thedefiectiona七poin七P. Choosethepositivedirectionsasupwardandtothe right.Choosepositivemomentsasclockwise. LMN=ON =RM,, (3.0m)十 (4448N)(4.6m) RM"=6820N [七othe1eft] エMM=ON =-RN"(3.0m)+(4448N)(4.6m) RN"=6820N [totheright] LFy=0N =RMy - 4448N RMν=4448N [upw乱rd] PPI.附 W押 i2p日sS.com Byinspection,thex-componentofforceinmemberMP isthesamemagnitudeandoppositedirectionasthe reactionatpointM. N 一 e U 一 0 0 一 2 M で 4 一 R I 一弘 一 十 一 イ 、 一 - m 一十 一 ﹀ d l u 一 勺G 広 明 一 向 4 1‘ , 一 円 U ﹁ │ 二 一 白 川 M 伝 山 、 必 R U v 剖 p i M M 一-6820N 8142N 3.0m ON P 一一 6820N N 6820N 4.6m ApplyingaunitloadatpointPproducesthefollowing virtualforces. M ....1.5N 1.8N 3開 O mI 0N P 1.5N N 1.5N 4.6m Thefollowingtablesummarizestheactualandvirtual forces. FQ member (N) MP 8142 NP 6820 MN 0 Fp (virtualforce) L (m) FQFpL (N-m) 1.8 1.5 0 5.49 80459 4.6 47058 3.0 0 total=127517
  • 18. Calcu1atethevertica1de丑ection抗 pointP. ムp=LFQ5L=玄FQ22 =占LFQFpL c川2却別ル一川Oω川 い 川O川 山 川1叩川O併6印刷叫叫)べ中(や1∞ x(2附 mm2 )(抗日 x(127517N-m)(1000空) TheanswerisA. 44.Fromthe1awsofequi1ibriumう eachreactionatan outersuppor七is ニ (t)((5:)州-18同 TheanswerisA. 45.Themaximumshearcanbedeterminedfroma sheardiagram. (A1thoughno七necessaryforthisprob- 1em,amomentdiagramhasa1sobeenconstructed,and thisisusedforsolvingthenextprob1em.) Thechangeinshearistheareaunderthe10addiagram uptothatpoint.Uptothecenterofthebe叩 1,pointA, Atpoin七A, ド 5.6N-(5:)(3m) V=-9.4N+18.75N = 9.35N (9.4N) Themomentisequa1totheareaunderthesheardia- gram. Pr自ctic告Ex自m2Soluti開 5 錦 Thechangeinmomenもisgivenbytheareaunder七he sheardiagramuptothatpoint. = (t)(5.6叩 2m) ル 3m =凡=1.12m 十(訪問問問m) ? ? ? x=1.12m 1.88m 1司 88m 3膚 1 3.1 M(N'm) 5.7 Fromthesheardiagram,themaximumva1ueforshear occurs剖 supportA. Vmax= 9.4N TheanswerisB. 46.Fromthemomentdiagramin801.45,theva1u巴for maximummomentoccursatsupportA. Mmax= [-5.7N.m[ = 5.7N-m TheanswerisD. PPIe附 w.ppi2p日sS.com
  • 19. 哩50 Appendix AftemoonSa円lpleExamination 42. 1nusingch10rinefordisinfection .a. concen住ationofHOC1ispH-dependent b. [HOCl]equals[OCllatpH7 c. e旺'ectivenessincreaseswithtimeofcontact d. totaleliminationofbacteriaispossible 43. A61・cm-diametercircularconcretesewerissetonaslopeof0.00l.The flowwhenitishalιfullis ajof伽 叫 制l b. 0.08m""/s c. 0.20m 3 /s d. 0.28m 3 /s 44. ArigidbarBCDissupportedbyahingedconnectionatsupportBand byacableACasshown.ThereactionforceatsupportBis A D B ド 2.0m 、、レO.5m....J a. 2.50kN b. 14.20kN c. 9.70kN d. 14.42kN 45. AsimplysupportedbeamABissubjectedtoauniformlydistributed loadandaconcentratedloadasshown.Themidspandef1ectionis 20kN ド 3.0m E=200GPa 1=3x106 m m 4 a. 16mm b. 38mm c. 22mm d. 8mm 『 ." 3 4 3.5 m Practice Exam III
  • 20. .. r--r・一一一一r- [}=[] SampleExam 15. 46. Asteelcolumn(W21x93)hasthebottomendsupportedrigidlysoas topreventbothrotationandtranslation.Thetopofthecolumnisfree totranslateandrotatein出eplaneofthediagrambutpinned(rotation allowed,translationprevented)intheperpendiculardirection.The lengthofthecolumnis32ft.Thecriticalbucklingloadasgivenby Euler'stheoryis a. 13.4kips b. 36.7kips c. 366kips d. 1002kips 47. Thethree-hingedarchABCcarriesauniformlydistributedloadfroma horizontaldeckasshown.百levalueofthehorizontalthrustatthesupportsis ー ド 15m 当6 10m オ a. 282kN b. 197kN c. 218kN d.252kN 48. Giventhetrussshown,theverticaldeflectionatBis 10抗 ド 8ft a. 1.2in b. 0.05in c. 0.2in d. 0.1in 当│ B E=29,000ksiforbothmembers A=2in2 forAB A=3in2 forBC 20kips 6 m 4 m
  • 21. 152 AppendixAftemoonSampleExamination 49. Themaximumbendingmomentforthebeamshownis ド 3.5m ~〈 1.5m a. 12.60kNm b. 8.82kNm c. 9.41kNm d. 10.25kNm 50. Aclientrequiresanewfaci1ityquicklytoreplaceonedestroyedbya hurricane.Heisnotcertainexactlywhatitshouldinclude.Thebest typeofcontracttobeusedin出issituationis a. unitprice b. lumpsum c. costplusfixedfee d. surety 51. Whatistheheight(白)ofaconicalspoilpile,if100ftbankcuydof commonearthwithanangleofreposeof320 anda12%swellis deposited? a. 9.6 b. 19.2 c. 10.4 d. 33.3 52. Determinethevolumeoffi1lforawholestation(100ft)ofcutwith theendareasshownbelow. ¥Al=120ft'/¥ι=150が/¥A'=100ft,/ a. 820yd3 b.456.8ydj c. 506.2yd J d.569.4yd J 53. ABCConstructionCompanyisthelowbidderat$500,000onabuilding project.Thecompanyhasprovidedthetypicalbidbond.Afteraward ofthecontract,thecompanydecidestowithdrawitsbid.Thesecond lowestbidwas$520,000.Howmuchisthesur~ty .company requiredto reimbursetheclientforthefailureofABCtosign? a. $500,000 b. $50,000 c. $20,000 d. $5000
  • 22. Solutions 唱63 41. d. Allofthestatementsareco町 ect. 42. a. 43. b. Qf=0.2m 3 /s Q/Qf=0.4 内 Q=0.08m'>/s 44. d. ThefreebodydiagramforBCDisshown.Note白atacommonmistake couldbe脳 出 国ngthatcableACformsa3-4-5凶anglewhenitd<∞sn't.百le ve同calandhorizontalcomponentsofT訂 eO.66Tand0.75T, respectively. T 10 Bx ヱMB=0.66Tx2-10x2.5=0司 T=ω 4 ヱぇ=久一0.75x18.94=0弓 Bx=14.20 ヱπ =久+0“x18.94-10=0=>By=-2.50 RB=石弓4 45. b. EI=r2xぽ型)X(3X1O-6m4)=6∞kNm2 ¥ m-J Deflectionatrnidspan(x=2m)duetopointloadisgivenby Pb 三 句 今 20xO.5 __, o=一一一[-x j +(Ll ーか)x]=_-~:. ~~:~ [-2j +(4l - OS)x2]=0.016 6LEr " 6x4xω。 Deflectionatrnidspan(x=2m)duetodistributedloadisgivenby δ 5wL 4 5x4x4 4 =一一=一一一一 =0.022 384EI 384x600 Totalmidspandeflection=0.038m=38mm 46. c. Bucklingaboutthestrong(x)鉱 is: K=2.0;L=32ft;rx=8.70in;KUr=88.3 Bucklingabouttheweak(y)axis: K=0.7;L=32ft;ry=1.84in;KUr=146.1 Solutions may be for problems with slightly different dimensions; I was given two apparently disjointed sets. One may conclude that problems are for the most part identical but with different dimensions.
  • 23. 唱64 Appendix AftemoonSampleExamination R π2EA π2x29,000x273 内 〆 " . = ナ ー ー す = ヲ =jOO阻 p: (子} 47. b. Therearefourextemalreactions-A_A.c..andC-andtwointemalx' "Y ~x' hingeforces-BxandBy一 旬 bedeterrnined.RecognizethatAxandCx areequalandopposite.SolvingforeitherAxorCxissufficient. TakingmomentsaboutA(entirestructure): ヱMA =500x山 +4ι -25Cy=0 TakingmomentsaboutB(righthalfofstructure): ヱMB.right=200x5-6に一以 =0 Solvingtheseequations,wegetCx=ー197.4kips. 48. d. Step1:UsingmethodofjointsatjointB: 凡B =+25.6k(T) FBC=ー16.0k(c) Step2:Calculatethememberforcesduetovirtualload(note:in出iscase, 出e毘 alloadandthevirtualloadlooksimilar,sowecanusescaling): Step3: んB=+1.28(T) fBC=-0.80(c) B =す rFfL1=~5似1.28 x(12.81x12)+-16x-O.8x(8xI2L=0.10血 """"'lAE) 2x29.000 3x29.000 49. b. Usingthefixedendmomentloadcasesin白eFESupplied-Rそference Handbookwiththefollowingdata: P=12kN;α=3.5m;b=1.5m;L=5m ....
  • 24. =-.: Solutions 唱65 Pab2 12x3.5x1.52 =ーで一 =--_._.__--_.- =3.78kNrn AB 72 L~ Pa2 b 12x3.52 x1.5=ーで一=--_._.__-._.-==8.82kNrn BA r2L~ 12 TakingrnornentsaboutA, ヱMA =+3.78一 以 3.5-8.82+玖 =0叫 =9.408釦 dAy=2.592 Thesheardiagrarnandbendingrnornentdiagrarns紅 e: 2.592 9.408 +5.292kNm -3.78kNm -8.82kNm 50. c. Costp1usfixedfee.Choicesaandbrequireadetaileddesign;choiced isabondingcornpany. 51. c. Loosecuft=100cuydx1.12x27cuft/cuyd=3024cuft Basediarneter(B)==(7.64x3024/tan320 )1/3=33.3ft Height==(33.3/2)(tan320 ) ==10.4ft 52. d. 百1isproblerncanbesolvedusingtheprisrnoidalrnethod.Vo1urne==(1 cuyd/27cuft)1∞ft[120sqft+(4x 150sqft)+100sqft]/6= 506.2cuyd 53. c. Theclientcanonlyrecoverthe10ssincurredbythedefault,whichis 出edi百erencebetweenthelowbidandthesecond10westbid:$20,000. 54. b. Totalcost=$50,000+$5000==$55,000 Unitcost==$55,000/200==$275.00/ftNote出attheunitpricernust includethecontractor'sprofit/rnarkup.
  • 25. Determinacy and Stability Determinacy and Stability I Trusses are statically determinate when all the bar forces can be determined from the equations of statics alone. Otherwise the truss is statically indeterminate. A truss may be statically/externally determinate or indeterminate with respect to the reactions (more than 3 or 6 reactions in 2D or 3D problems respectively). A truss may be internally determinate or indeterminate. If we refer to j as the number of joints, R the number of reactions and m the number of members, then we would have a total of m + R unknowns and 2j (or 3j) equations of statics (2D or 3D at each joint). If we do not have enough equations of statics then the problem is indeterminate, if we have too many equations then the truss is unstable. 2D 3D Static Indeterminacy External R > 3 R > 6 Internal m + R > 2j m + R > 3j Unstable m + R < 2j m + R < 3j Victor E. Saouma; CVEN 3525; Univ. of Colorado Reactions 5/23 TRUSSES
  • 26. Method of Joints Method of Joints II This method should be used when all member forces must be determined. In truss analysis, there is no sign convention. A member is assumed to be under tension (or compression). If after analysis, the force is found to be negative, then this would imply that the wrong assumption was made, and that the member should have been under compression (or tension). On a free body diagram, the internal forces are represented by arrow acting on the joints and not as end forces on the element itself. That is for tension, the arrow is pointing away from the joint, and for compression toward the joint. cc t C A B -ve +ve -ve A B C Victor E. Saouma; CVEN 3525; Univ. of Colorado Reactions 9/23
  • 27. Example; Method of Joints Example; Method of Joints I A B C D E F G H 12 8 20 k 40 k 40 k 10' 32' 24' 24' 24' 24' 1 R = 3, m = 13, 2j = 16, and m + R = 2j √ 2 We compute the reactions +   ¡ ΣME z = 0; ⇒ (20 + 12)(3)(24) + (40 + 8)(2)(24) + (40)(24) − RAy (4)(24) = 0 ⇒ RAy = 58 k 6 + ?ΣFy = 0; ⇒ 20 + 12 + 40 + 8 + 40 − 58 − REy = 0 ⇒ REy = 62 k 6 Victor E. Saouma; CVEN 3525; Univ. of Colorado Reactions 10/23
  • 28. Example; Method of Joints Example; Method of Joints II 3 Consider each joint separately: Node A: Clearly AH is under compression, and AB under tension. 58 k FAB FAH A + 6ΣFy = 0; ⇒ −FAHy + 58 = 0 FAH = l ly (FAHy ) ly = 32; l = 322 + 242 = 40 ⇒ FAH = 40 32 (58) = 72.5 k Compression +- ΣFx = 0; ⇒ −FAHx + FAB = 0 FAB = Lx Ly (FAHy ) = 24 32 (58) = 43.5 k Tension Node B: Victor E. Saouma; CVEN 3525; Univ. of Colorado Reactions 11/23
  • 29. Example; Method of Joints Example; Method of Joints III B 20 k FBH 43.5 k FBC +- ΣFx = 0; ⇒ FBC = 43.5 k Tension + 6ΣFy = 0; ⇒ FBH = 20 k Tension Node H: H 12 k FHG FHC 72 k 20 k FHCx FHCy FAHy FHGx FHGy FAHx Victor E. Saouma; CVEN 3525; Univ. of Colorado Reactions 12/23
  • 30. Example; Method of Joints Example; Method of Joints IV +- ΣFx = 0; ⇒ FAHx − FHCx + FHGx = 0 43.5 − 24 242+322 (FHC ) + 24 242+102 (FHG) = 0 + 6ΣFy = 0; ⇒ FAHy + FHCy − 12 + FHGy − 20 = 0 58 + 32 242+322 (FHC ) − 12 + 10 242+102 (FHG) − 20 = 0 This can be most conveniently written as 0.6 −0.921 −0.8 −0.385 FHC FHG = 43.5 26.0 Solving we obtain FHC = −7.5 and FHG = −52, thus we made an erroneous assumption in the free body diagram of node H, and the final answer is FHC = 7.5 k Tension FHG = 52 k Compression Node E: Victor E. Saouma; CVEN 3525; Univ. of Colorado Reactions 13/23
  • 31. Example; Method of Joints Example; Method of Joints V E 62 k FED FEF ΣFy = 0; ⇒ FEFy = 62 ⇒ FEF = 242+322 32 (62) = 77.5 k C ΣFx = 0; ⇒ FED = FEFx ⇒ FED = 24 32 (FEFy ) = 24 32 (62) = 46.5 k T The results of this analysis are summarized below Victor E. Saouma; CVEN 3525; Univ. of Colorado Reactions 14/23
  • 32. Example; Method of Joints Example; Method of Joints VI 43.5 43.5 46.5 46.5 72.5 20 32 2.5 7.5 4 0 77.5 46.5 6258 43.5 52 52 58 62 12 8 20 40 40 4 We could check our calculations by verifying equilibrium of forces at a node not previously used, such as D Victor E. Saouma; CVEN 3525; Univ. of Colorado Reactions 15/23
  • 33. Examples Beams Example Beam; 1 A B C D E 2 k/ft 4ft 4ft 4ft6ft 4 3 11 k 10 k Reactions are determined from the equilibrium equations (+ ) ΣFx = 0; ⇒ −Ax + 6 = 0 ⇒ Ax = 6 k (+  ¡) ΣMA = 0; ⇒ (11)(4) + (8)(10) + (4)(2)(14 + 2) − Ey (18) = 0 ⇒ REy = 14 k (+ 6) ΣFy = 0; ⇒ Ay − 11 − 8 − (4)(2) + 14 = 0 ⇒ Ay = 13 k Victor E. Saouma; CVEN 3525; Univ. of Colorado Internal Forces 11/31 SHEAR AND MOMENT DIAGRAMS
  • 34. Examples Beams Example Beam; 2 A B C D E 2 k/ft 4ft 4ft 4ft6ft 4 3 11 k 10 k Shear are determined next. 1 At A the shear is equal to the reaction and is positive. 2 At B the shear drops (negative load) by 11 k to 2 k. 3 At C it drops again by 8 k to −6 k. 4 It stays constant up to D and then it decreases (constant negative slope since the load is uniform and negative) by 2 k per linear foot up to −14 k. 5 As a check, −14 k is also the reaction previously determined at F. Moment is determined last: 1 The moment at A is zero (hinge support). 2 The change in moment between A and B is equal to the area under the corresponding shear diagram, or ∆MB−A = (13)(4) = 52. 3 etc... Victor E. Saouma; CVEN 3525; Univ. of Colorado Internal Forces 12/31
  • 35. Examples Beams Example Beam; 3 A B C D E 2 k -6 k -14 k Slope= dV/dx=w=-2 dM/dx=+2 dM/dx=-6 0+52=52 52+12=64 64-24=40 10 k 11 k 2 k13 k 8 k 2 k A B C E 13 k A=(13)(4)=52 A=(6)(2)=12 A=(-6)(4)=-24 A=-4(6+14)/2=-40 -6 k Victor E. Saouma; CVEN 3525; Univ. of Colorado Internal Forces 13/31
  • 36. arches Three Hinged Arch; Point Loads Three Hinged Arch; Point Loads I 80' 60' 33.75' 30' 20 k B 80 k 26.25' A C 20' 20' 20 kB 30 k 80 k A C HA VA VC HC VB VB HB HB 30 k Four unknowns, three equations of equilibrium, one equation of condition ⇒ statically determinate. +   ¡ ΣMC z = 0; (RAy )(140) + (80)(3.75) − (30)(80) − (20)(40) + RAx (26.25) = 0 ⇒ 140RAy + 26.25RAx = 2.900 + - ΣFx = 0; 80 − RAx − RCx = 0 + 6ΣFy = 0; RAy + RCy − 30 − 20 = 0 +  ¡ ΣMB z = 0; (Rax )(60) − (80)(30) − (30)(20) + (RAy )(80) = 0 ⇒ 80RAy + 60RAx = 3, 000 (22) Victor E. Saouma; CVEN 3525; Univ. of Colorado Cables Arches 19/24 ARCHES
  • 37. arches Three Hinged Arch; Point Loads Three Hinged Arch; Point Loads II Solving those four equations simultaneously we have:     140 26.25 0 0 0 1 0 1 1 0 1 0 80 60 0 0        RAy RAx RCy RCx    =    2, 900 80 50 3, 000    ⇒    RAy RAx RCy RCx    =    15.1 k 29.8 k 34.9 k 50.2 k    (23) We can check our results by considering the summation with respect to B from the right: +   ¡ ΣM B z = 0; −(20)(20) − (50.2)(33.75) + (34.9)(60) = 0 √ (24) Victor E. Saouma; CVEN 3525; Univ. of Colorado Cables Arches 20/24
  • 38. Examples Beam Example: Beam I Determine the deflection at point C. E = 29, 000 ksi, I = 100 in4 . 2 k/ft A A B B C C 15 k 45 k 0.5 k 1.5 k 1 k 20' 10' C -x-0.5x 15x-x2 -x2 Real Moment Virtual Moment x x Element x = 0 M δM AB A 15x − x2 −0.5x BC C −x2 −x Victor E. Saouma; CVEN 3525; Univ. of Colorado Virtual Work 10/23 BEAM DEFLECTION, VIRTUAL FORCE
  • 39. Examples Beam Example: Beam II Applying the principle of virtual work, we obtain ∆CδP δW ∗ = L 0 δM(x) M(x) EIz dx δU ∗ (1)∆C = 20 0 (−0.5x) (15x − x2) EI dx + 10 0 (−x) −x2 EI dx = 2, 500 EI ∆C = (2, 500) k − ft3 (1, 728) in3 / ft3 (29, 000) ksi(100) in4 = 1.49 in Victor E. Saouma; CVEN 3525; Univ. of Colorado Virtual Work 11/23
  • 40. Examples Truss; Simple Truss; Simple I Determine the deflection at node 2 for the truss. 12' 1 1 3 4 4 7 5 2 3 5 60 k 120 k 6 12' 12' A=5.0 in2 each; E=10x103 ksi 60 120 -117.3-83.8 37.5 52.5 16.8 -16.8 -45.0 75.0 105.0 A -0.50 -0.56 -0.56 0.25 0.25 0.56 0.56 0.5 0.51.0 -0.54 -1.124 - 1.574 0.45 0.63 0.226 -0.226 δP (e) P(e), L, A, E, δP (e) P(e) L AE Member kips kips ft in2 ksi 1 +0.25 +37.5 12 5.0 10 × 103 +22.5 × 10−4 2 +0.25 +52.5 12 5.0 10 × 103 +31.5 × 10−4 3 -0.56 -83.8 13.42 5.0 10 × 103 +125.9 × 10−4 4 +0.56 +16.8 13.42 5.0 10 × 103 +25.3 × 10−4 5 +0.56 -16.8 13.42 5.0 10 × 103 −25.3 × 10−4 6 -0.56 -117.3 13.42 5.0 10 × 103 +176.6 × 10−4 7 -0.50 -45.0 12 5.0 10 × 103 +54.0 × 10−4 +410.5 × 10−4 Victor E. Saouma; CVEN 3525; Univ. of Colorado Virtual Work 18/23 You will not get something as complex as this truss TRUSS-DEFLECTION; Virtual force
  • 41. Examples Truss; Simple Truss; Simple II The deflection is thus given by δP∆ = 7 1 δP (e) PL AE ∆ = (410.5 × 10−4 )(12 in/ ft) = 0.493 in Victor E. Saouma; CVEN 3525; Univ. of Colorado Virtual Work 19/23
  • 42. Examples Truss with initial camber Truss with initial camber I It is desired to provide 3 in. of camber at the center of the truss shown below 1 32 6 @ 27' 1 k 36' by fabricating the endposts and top chord members additionally long. How much should the length of each endpost and each panel of the top chord be increased? Assume that each endpost and each section of top chord is increased 0.1 in. Member δP (e) int ∆L δP (e) int ∆L 1 +0.625 +0.1 +0.0625 2 +0.750 +0.1 +0.0750 3 +1.125 +0.1 +0.1125 +0.2500 Victor E. Saouma; CVEN 3525; Univ. of Colorado Virtual Work 22/23 You may get somethin g along those lines, or with temperat ure load
  • 43. Examples Truss with initial camber Truss with initial camber II Thus, (2)(0.250) = 0.50 in Since the structure is linear and elastic, the required increase of length for each section will be 3.0 0.50 (0.1) = 0.60 in If we use the practical value of 0.625 in., the theoretical camber will be (6.25)(0.50) 0.1 = 3.125 in Victor E. Saouma; CVEN 3525; Univ. of Colorado Virtual Work 23/23
  • 44. V. Saouma; CVEN‐3525 1 Solve for the vertical displacement at C of the following structure. Step 1. Draw free body diagram and calculate all forces due to real load and virtual load . A B C 80 160 D 2k yD xD xA yA (CCW +ve) 0 (60) 2(240) 0 0 0 0 2 8 0 (1) 8 A x x x x y y y x x M D F D A F A D D k A k + Σ = − = →Σ = − + = + ↑ Σ = − = = − + = A B C 2k 80 160 3 4 D 2 7 Aluminum rod, A =1 in ,E =10 psi 2 4 7 Aluminum Beam, A =10 in , I = 171 in ,E =10 psi 60 REACTIONS AND DISPLACEMENTS
  • 45. V. Saouma; CVEN‐3525 2 D yD 8k 3 4 T 3 (2) 5 4 1 5 0 8 y x T D T k D k T = = = = .(2) From .( 4 1) 6 A y y From Eq Eq D k k = = Likewise, we can get the reaction due to unit virtual load at point C by similar manner. A B C 80 160 2k6k 8k 8k 4k A B C 80 160 1Pδ =3k 4k 4k 2k Free Body Diagram due to real load Free Body Diagram due to virtual load 160”
  • 46. V. Saouma; CVEN‐3525 3 Element x=0 P δP DB D 10 5 Step 2. Determine forces in cable and beam for each elements due to real load and virtual  load (i) Cable.  Tension is constant along the cable length.  (ii)    Beam.  A B C 80 160 2k6k 8k 8k 4k A B C 80 160 1Pδ =3k 4k 4k 2k Free Body Diagram due to real load Free Body Diagram due to virtual load Element x=0 P δP M δM AB A ‐8 ‐4 ‐4x ‐2x BC B 0 0 2x‐320 x‐160 You may not get something as complex for deflection; however make sure that you understand this.
  • 47. V. Saouma; CVEN‐3525 4 From (i) and (ii) * * 100 80 0 0 80 160 0 0 100 80 2 7 2 7 0 0 (5)(10) ( 4)( 8) (1) ( 2 )( 4 ) ( 160)(2 320) (5)(10) ( 4)( 8) (1 )(10 ) (10 )(10 ) ( 2 C L L w u C c b b b P M P P dx M dx AE EI dx dx A E A E x x x x dx EI EI dx dx in psi in psi x δ δ δ δ δΔ = + − − Δ = + − − − − + + − − = + − + ∫ ∫ ∫ ∫ ∫ ∫ ∫ ∫ 80 160 7 4 7 4 0 0 )( 4 ) ( 160)(2 320) (10 )(171 ) (10 )(171 ) 0.0005 0.0000256 0.0007984 0.001597 0.0005256 0 0 . . 00 00 239 ) 54 29 ( C Axial Contribution Flexure Contrib t C u ion x x x dx psi in p i i n n i s − − − + Δ = + + + = + Δ = ↓ ∫ ∫
  • 49. V. Saouma; CVEN‐3525 11 (a). Determine the horizontal deflection at C. Assume AE constant. A E D C B 600 lb 1500 lb 8' 8' 8' 2 1 5 6 4 3 A E D C B 1Pδ = 8' 8' 8' 2 1 5 6 4 3 Real load Virtual load Member δP(lb) P(lb) L(in) AE δP.(PL/AE) 1 2 1,200 96 AE 230,400/AE 2 2 1,200 96 AE 230,400/AE 3 0 ‐1,500 48 AE 0 4 ‐2.24 ‐1,341.64 107.33 AE 322,560/AE 5 0 1,677.05 107.33 AE 0 6 ‐2.24 ‐3,018.69 107.33 AE 725,759.6/AE 1,509,119lb in AE − ∑ = 6 1.51 10 ( )h lb in C AE × − Δ = → You will not get such a complex truss, however the procedure is very important to understand
  • 50. V. Saouma; CVEN‐3525 12 (b). Remove the loads and determine the horizontal displacement of C if members AB and  BC  experience a temperature increase ΔТ=200°F, Take A = 2 in2, E = 29,000 ksi, and α=10‐6/°F Member L(in) α(1/°F) ΔT(°F) ΔL=αΔTL(in) δP δP.ΔL 1 96 10‐6 200 0.0192 2 0.0384 2 96 10‐6 200 0.0192 2 0.0384 3 48 10‐6 0 0 0 0 4 107.33 10‐6 0 0 ‐2.24 0 5 107.33 10‐6 0 0 0 0 6 107.33 10‐6 0 0 ‐2.24 0 0.0768∑ = 0.0768 ( )h C inΔ = → A E D C B 2 1 5 6 4 3200T FΔ = You will not get such a complex truss, however the procedure is very important to understand
  • 51. V. Saouma; CVEN‐3525 13 c. Remove the loads and determine the horizontal displacement of C if member CD is  fabricated 0.5 in too short. A E D C B 2 1 5 6 4 3 Fabricated 0.5 in  too short. Member L(in) ΔL(in) δP δP.ΔL 1 96 0 2 0 2 96 0 2 0 3 48 0 0 0 4 107.33 ‐0.5 ‐2.24 1.12 5 107.33 0 0 0 6 107.33 0 ‐2.24 0 1.12∑ = 1.12 ( )h C inΔ = → You will not get such a complex truss, however the procedure is very important to understand